MBE Missed Q's
A man from a foreign country obtained a doctorate in political science from a state university and applied to teach there. The man was denied employment at the university under a state law requiring all teachers within the state to be United States citizens. Is the state's citizenship requirement constitutional as it applies to the man? response - incorrect A Yes, because states have the right to set minimal standards for state employees under the Tenth Amendment. B Yes, because a university political science teacher would exert a great deal of influence over the attitudes of students toward government, the political process, and citizenship. C No, because the citizenship requirement is not rationally related to a legitimate state interest. D No, because the citizenship requirement is not necessary to achieve a compelling state interest.
A state generally may not discriminate against aliens absent a compelling state interest, and no compelling interest is served by prohibiting aliens from teaching at a state university. (A) is incorrect. The Tenth Amendment reserves to the states power not granted to the federal government. The Constitution vests the power to regulate aliens in Congress, and thus the states do not have power to control aliens under the Tenth Amendment. (B) is incorrect because it states the standard that the Supreme Court has applied to primary and secondary school teachers. The Supreme Court has upheld state statutes prohibiting aliens from teaching primary or secondary school on the rationale that teachers at the elementary and high school level have a great deal of influence over the attitudes of young students toward government, the political process, and citizenship. It is doubtful that the Court would extend this rationale to university teachers. (C) is incorrect because it states the wrong standard. If state discrimination against aliens relates to participation of aliens in the functioning of state government, the rational basis test applies. Merely teaching political science at a state university is not equivalent to participating in the political process.
A credit card company filed a civil action against a consumer in federal district court seeking to recover the unpaid balance on the consumer's account. The credit card company properly served process on the consumer, but the consumer failed to file or serve a timely answer to the complaint. The clerk of court on motion of the credit card company made an entry of default, but default judgment has not been entered. On which grounds may the court set aside the entry of default? response - incorrect A For the same limited grounds for which any court judgment may be set aside. B Only if the court finds that the consumer could not have filed and served a timely answer despite using reasonable diligence. C If the consumer demonstrates that there was good cause for his failure to file and serve a timely answer and that he has a viable defense. D On any grounds that the court, in its discretion, finds just.
An entry of default may be set aside for "good cause shown." Although not specifically required by the Federal Rules, a majority of courts also will require some showing of a meritorious defense. (A) is incorrect because the grounds are not limited to the grounds required for setting aside regular court judgments. (B) is incorrect because that answer is also too limiting on the concept of "good cause shown." For example, a default might result from an honest mistake of the attorney, but the entry of default may nonetheless be set aside if the attorney acts promptly to correct the mistake. (D) is incorrect because a majority of courts will also require a showing of a meritorious defense.
Shortly after a professor at a state university completed her second year of teaching, she was informed that her contract was not being renewed for the following year. By state law, a professor does not acquire tenure until after she has completed three consecutive years of teaching. Before acquiring tenure, state law does not require either a statement of reasons or a hearing when a professor's contract is not renewed, and the university administration refused to give either to the professor. Which of the following, if established, sets forth the strongest constitutional argument that the professor could make to compel the university to furnish her a statement of reasons for the failure to rehire her and an opportunity for the hearing? response - incorrect A She purchased a home in anticipation of renewal of her contract, because most professors who had taught two years were rehired. B She had been voted the most popular professor on campus in each of her first two years of teaching. C She was the only teacher at the university whose contract was not renewed that year. D There is evidence to indicate that the decision not to rehire the professor was not based on her ability to teach.
Answer Discussion - Incorrect The strongest argument the professor could make is that the decision was not based on her ability to teach. The professor is an at-will employee, and under most circumstances may be discharged "for any reason or no reason at all." Thus, normally, evidence regarding the motives for dismissal is irrelevant. The question here, however, is what the strongest argument is that the professor could make, and (D) creates at least an inference that an impermissible motive might be present (gender, free speech, etc.). (A) is a weaker answer because the professor has no property interest in continued employment; a mere expectation of continued employment is not enough, even when coupled with reliance (her buying a house). There must be a legitimate claim or entitlement—created by a contract or clear policy—that employment can be terminated only for cause. The bases alleged in (B) are arguably irrelevant; the professor's popularity may or may not have anything to do with her ability, and even if it does, she remains an at-will employee. (C) might under some circumstances offer an argument, but there could be any number of valid explanations for keeping others and letting a particular professor go, including budget constraints, subject needs, etc. (D) is, accordingly, the strongest of the possibilities.
A police officer stopped a boyfriend and girlfriend on a college campus under suspicion that they were selling illegal drugs out of their backpacks. The officer asked the two some questions and then asked if he could look in their backpacks. They consented to the search, but the officer found nothing suspicious in either backpack. The officer then patted them both down and found a package of drugs taped to the boyfriend's chest. The boyfriend and the girlfriend were charged with possession. The boyfriend pleaded guilty to possession. Prior to trial, the girlfriend's attorney moved to suppress the introduction of the drugs at her trial. At the suppression hearing, the attorney stipulated that the girlfriend owned the drugs. How should the court rule on the motion? response - incorrect A In favor, because the girlfriend was not in possession of the drugs. B In favor, because both the boyfriend and the girlfriend had a reasonable expectation of privacy when the officer elected to search them. C Against, because the girlfriend has stipulated that she owns the cocaine. D Against, because the search of the boyfriend's body did not violate the girlfriend's reasonable expectation of privacy.
Because the girlfriend's Fourth Amendment rights were not violated by the unlawful search of the boyfriend, the drugs may be introduced against the girlfriend at trial. Under Rakas v. Illinois (1978), Fourth Amendment rights may be enforced by the exclusion of evidence only at the instance of one whose own protection was infringed by the search and seizure. Ownership of the property seized does not automatically establish violation of one's reasonable expectation of privacy; it is just one factor in the totality of the circumstances that the court will consider. [Rawlings v. Kentucky (1980)] Here, the drugs were seized from the boyfriend as a result of a search that may have violated his Fourth Amendment rights. However, nothing in the question indicates that the girlfriend had a reasonable expectation of privacy in the boyfriend's body (such as the right to exclude others from searching the boyfriend if the boyfriend had consented). Under the circumstances in this case, the girlfriend's ownership of the drugs does not establish a reasonable expectation of privacy with regard to the search of the boyfriend. (A) is incorrect because the fact that the girlfriend did not have actual possession of the drugs does not require that the drugs be excluded from her trial. She may be liable under the jurisdiction's possession statute if she had sufficient dominion or control over the drugs to be in constructive possession of it. (B) is incorrect even though it is a true statement. The officer's search of the girlfriend may have violated the girlfriend's reasonable expectation of privacy, but no evidence was obtained by an illegal search of the girlfriend. The officer's search of the boyfriend may have violated the boyfriend's reasonable expectation of privacy but not the girlfriend's expectation of privacy (as discussed above). Because the girlfriend's Fourth Amendment rights were not violated by the search of the boyfriend, she cannot use the exclusionary rule to suppress introduction of the drugs. (C) is incorrect because the defendant has the right to testify and stipulate to facts at a suppression hearing without her testimony or stipulation being admitted against her at trial on the issue of guilt. [Simmons v. United States (1968)] This rule allows a defendant to assert a possessory or ownership interest in illegally seized evidence just for purposes of invoking the exclusionary rule; if she fails to have the evidence excluded, she may still deny possession or ownership at trial.
A small processor of specialized steel agreed in writing with a small manufacturer of children's toys that it would supply, and the manufacturer would buy, all of the manufacturer's specialized steel requirements over a period of years at a set price per ton of steel. Their contract did not include a nonassignment clause. Recently, the toy manufacturer decided to abandon its line of steel toys, so it made an assignment of its rights and delegation of its duties under the contract to a toymaker many times larger. The large toymaker notified the steel processor of the assignment and relayed to the processor its good faith belief that its requirements will approximate those of the assignor. Must the steel processor supply the requirements of the large toymaker? response - incorrect A Yes, because there was no nonassignment clause in the contract. B Yes, because the large toymaker acted in good faith to assure the steel processor that its requirements will approximate those of the small manufacturer into whose shoes it stepped. C No, because requirements contracts are not assignable under the UCC D No, because the steel processor did not give prior approval of the assignment.
Because the large toymaker acts in good faith in setting its requirements to approximately those of the small manufacturer into whose shoes it stepped, the contract may be assigned. The contract in this question is a "requirements" contract: The steel processor must sell the small manufacturer of children's toys all the specialized steel it requires for its toys. Generally, the right to receive goods under a requirements contract is not assignable because the obligor's duties could change significantly. In fact, here, a significant change would seem possible because the large toymaker is a larger company than the small manufacturer and its needs could be greater. However, the UCC allows the assignment of requirements contracts if the assignee acts in good faith not to alter the terms of the contract. [UCC �2-306] (The UCC applies here because goods are involved.) Thus, assuming the large toymaker's requirements remain about the same as the small manufacturer's requirements, the steel processor would be required to honor its contract, now assigned to the large toymaker. (A) is wrong because requirements contracts may be nonassignable, even without a nonassignment clause. Thus, the clause would be irrelevant. The only thing that could allow assignment of a requirements contract is a good faith limitation, as addressed in choice (B). (C) is wrong because the UCC does allow requirements contracts to be assigned, as long as the good faith limitation is satisfied. (D) is similarly incorrect. The UCC would allow assignment without approval by the obligor if there is a good faith limitation on the requirements.
A local news station broadcast a live interview with a bystander about his views concerning the state of local education. The bystander responded by saying that the principal of his daughter's high school had been embezzling school funds for years. The principal saw the telecast and also recorded it. He sued the owner of the station for defamation. At trial, the principal sought to testify to the defamatory statement made in the interview. Will the principal's testimony likely be held to be admissible? response - incorrect A No, because the testimony would be hearsay not within any exception. B No, because a recording of the interview exists. C Yes, because the statement is being offered to show its effect on the principal. D Yes, because the principal personally saw the interview on television.
Because the principal had firsthand knowledge that the statement was made, his testimony will be admissible unless there is a specific rule excluding the evidence. Witnesses are generally presumed competent to testify until the contrary is demonstrated. While a witness may not testify to a matter unless evidence is introduced to support a finding that the witness has personal knowledge of the matter, this evidence may consist of the witness's own testimony. (A) is incorrect. Hearsay is a statement, other than one made by the declarant while testifying at the trial or hearing, offered in evidence to prove the truth of the matter asserted. In a defamation action, evidence of the statement alleged to be defamatory is not hearsay because the evidence is by definition not offered to prove the truth of the matter asserted. It is offered only to show that the actionable statement was made. (B) is incorrect. Because the principal had firsthand knowledge of the event he can testify about the event, even though there might exist a recording that would be better proof of the event. The "best evidence rule" does not apply because the recording is not an essential repository of the facts recorded. (C) is incorrect. Although the statement is not hearsay, it is not being offered to show its effect on the hearer (e.g., knowledge, motive), but rather to show that the statement was made, as explained above.
A competitor of a restaurant obtained the restaurant's secret recipe for fried chicken and began using it. The restaurant commenced a diversity action in federal court against the competitor for misappropriation of trade secrets. After the pleadings were served, but before discovery was commenced, the restaurant moved for a preliminary injunction to prevent the competitor from using the recipe. In support of its motion, the restaurant submitted an affidavit of a former employee of the restaurant, who admitted that he stole the recipe from the restaurant and sold it to the competitor. The court granted the restaurant's motion. May the competitor immediately appeal the court's decision? response - incorrect A No, because the court's decision was based on substantial evidence. B No, because the grant of a motion for a preliminary injunction is not a final order. C Yes, because the grant of a preliminary injunction is immediately appealable as of right. D Yes, because the court should have allowed the competitor to depose the former employee before granting the motion.
C) The competitor may immediately appeal. A party may appeal as of right any order granting, continuing, modifying, refusing, dissolving, or refusing to dissolve or modify an injunction. (B) is incorrect because, although generally only final orders are appealable, there are exceptions for certain orders, including orders granting injunctions. (A) is incorrect because this choice states a standard of review on appeal, not a basis for an appeal. (D) misconstrues the question. At best, this reason states grounds for asking the appeals court to vacate the injunction. The court's order is appealable because the grant of the injunction is appealable as of right. The grounds for the appeal are not pertinent.QUESTION ID: MJ194
A woman was struck by a brick with her name scrawled on it that was thrown through her bedroom window. The victim believes that her ex-boyfriend, who is a gang member, threw the brick because she has become active in anti-gang groups, but she did not actually see him throw it. If the ex-boyfriend is arrested and put on trial for battery, which of the following items of the victim's proposed testimony is LEAST likely to be admitted? response - incorrect A The victim recently moved to a new apartment and only her ex-boyfriend and a few family members knew its location. B The victim had testified against a member of her ex-boyfriend's gang last month in a drug case. C On another occasion, the victim had seen her ex-boyfriend throw a rock through the window of a rival street gang member. D Immediately after the brick went through her window, the victim heard a voice she recognized as her ex-boyfriend's yell, "If you don't start minding your own business, you'll get a lot worse than this next time!"
Evidence of the defendant's other crimes or misconduct is admissible only if relevant to some issue other than the defendant's character or propensity to commit the crime charged. Such acts would be admissible to show motive, intent, absence of mistake, identity, or a common plan or scheme. Of these, the only one possibly relevant to these facts is identity. Evidence that the accused committed prior criminal acts that are so distinctive as to operate as a "signature" may be introduced to prove that the accused committed the act in question. Merely throwing an object, such as a brick, through a window could not be considered so distinctive as to operate as a signature. Thus, this evidence would not show identity. The only possible reason for offering the evidence is to show the ex-boyfriend's propensity to commit the crime charged, in which case the testimony will be inadmissible. (A) is wrong because it is circumstantial evidence that the ex-boyfriend threw the brick. It is relevant because it tends to make it more probable that he threw the brick than it would be without the evidence. (B) is wrong because it is relevant and goes to motive. It too makes it more probable that the ex-boyfriend threw the brick than it would be if the victim had not testified against a member of his gang. (D) is wrong because the victim's identification of the ex-boyfriend's voice places him at the scene and is thus relevant. It is more probable that he threw the brick than it would be in the absence of this testimony. The identification of a voice is properly authenticated by the opinion of a person familiar with the alleged speaker's voice. As his ex-girlfriend, the victim would be sufficiently familiar with the ex-boyfriend's voice to make a proper identification.
After a sporting event at a stadium, one of the fans sought out the referees to complain about their handling of the game. The fan took out an electronically amplified bullhorn and knocked on the door of the referees' room. When one of the referees opened it, the fan began yelling and berating the referee through the bullhorn. The referee slammed the door shut, striking the bullhorn and jamming it against the fan's mouth, knocking out two of his teeth. If the fan asserts a claim based on battery against the referee and the referee prevails, what is the likely reason? response - incorrect A The referee did not foresee that the bullhorn would knock out the fan's teeth. B The referee did not know that the door was substantially certain to strike the bullhorn. C The referee was entitled to use force to protect himself. D The fan's conduct provoked the referee's response.
If the referee prevails, it will be because he did not know the door would strike the bullhorn, so he did not have the intent to commit a battery. Battery requires: (i) an act by defendant that causes a harmful or offensive contact to plaintiff's person; (ii) intent to cause the harmful or offensive contact; and (iii) causation. Here, there was a harmful contact caused by the referee. The only consideration is whether the referee had the requisite intent. If a person knows with substantial certainty the consequences of his action, he has the intent necessary for this type of tort. If the referee did not know that the door was substantially certain to hit the bullhorn the fan was holding, the referee did not have the intent necessary for battery. (For purposes of battery, anything connected to or being held by the plaintiff is usually considered part of the plaintiff's person.) (A) is wrong because if the referee intended to cause a harmful contact (a battery), he is liable for all of the consequences of his actions, whether he intended them or not. A defendant need not foresee the extent of the injuries caused by his intentional act to be held liable for them. (C) is wrong because this is not a case of self-defense. Self-defense is appropriate when a person reasonably believes that he is being or is about to be attacked. Nothing in the facts shows any basis for the referee to believe that the fan was going to harm him. Thus, self-defense is not appropriate here. (D) is wrong because it does not provide the referee with a defense. The fan's conduct angered the referee and may have triggered his actions, but because the fan's conduct was not sufficient to allow the referee to act in self-defense, the referee's use of force here is not excused.
Adherents of a particular religion whose tenets focused mostly on business practices forbade women from studying their sacred texts. A group of college students who were adherents of that religion applied to use an empty room at their state college to study sacred texts. The school permitted numerous student groups to use its facilities for extracurricular activities during times when classes were not in session. However, the school administration denied the requests from the group in question, claiming that it would be in violation of a state statute forbidding any group using public facilities to discriminate on the basis of race or gender. The students brought an action in federal court challenging application of the statute to them by the school administration. If the court finds the actions of the school valid, what is the most likely reason? response - incorrect A Permitting the religious group to hold the meeting in a public school facility would violate the Establishment Clause, applicable to the state under the Fourteenth Amendment. B The statute is the least restrictive means of advancing the state's compelling interest in ending discrimination by groups using public facilities. C Allowing student groups to use classroom facilities when classes are not in session does not constitute state action for purposes of the Fourteenth Amendment. D The right of freedom of association does not apply to groups involved in business and commercial activities.
If the school's action is valid, it will be because the state statute is the least restrictive means of advancing the state's compelling interest in ending discrimination by groups using public facilities. While schools are generally not public forums, they may become a designated public forum by being held open to student groups for meetings. In that case, the First Amendment may be violated if a college restricts use of its classrooms based on the content of a student group's speech. To justify content-based regulation of otherwise protected speech, the government must show that the regulation is necessary to achieve a compelling state interest that cannot be satisfied by less restrictive means. Similarly, the right to associate for expressive purposes is not absolute. At the very least, the right may be infringed to serve a compelling government interest, unrelated to the suppression of ideas, that cannot be achieved through means significantly less restrictive of associational freedoms. Here, the state's interest in not allowing its facilities to be used by groups practicing discrimination of various types is compelling. [See Roberts v. United States Jaycees (1984)] The denial of access to the student group based on the students' religious principles, while it may be viewed as content-based discrimination, is the most narrowly drawn means of advancing the state's interest. [See Bob Jones University v. United States (1983)] (A) is incorrect because a school does not violate the Establishment Clause by permitting a religious student group the same after-class access to its facilities that other student groups have. [Good News Club v. Milford Central School (2001)] (C) is incorrect because the actions of administrators of a state college in allowing or denying access to its facilities is clearly state action that brings the Fourteenth Amendment into play. (D) is incorrect. While the right to join together for expressive or political activity, which is protected by the First Amendment, may be less strong for large organizations that engage in both commercial and expressive activity than for smaller and more selective groups, it is still a recognized right. [See Roberts v. United States Jaycees, supra] Furthermore, the student group's discussion of business activity in this case is tied to its religion. Hence, the association rights of the student group are based on freedom of religion as well as freedom of expression. The state would probably have to show a compelling interest to support a restriction on the group's association rights.
A state bans the use of disposable diapers to reduce the volume of nonbiodegradable material in its landfills. The ban was a boon for diaper services within the state, but many parents of young children were displeased with the use of conventional diapers. With support from retail establishments that lost business from the disposable diaper ban, a grass roots coalition formed to fight the ban funded a study showing that the trucks and cleaning supplies used by diaper services within the state harmed the environment more than disposable diapers. The coalition and retailers then filed suit seeking to have the ban on disposable diapers declared unconstitutional. If the court strikes down the statute, on which of the following constitutional provisions would its decision most likely be based? response - correct A The Equal Protection Clause of the Fourteenth Amendment. B The Due Process Clause. C The Impairment of Contracts Clause. D The Privileges or Immunities Clause of the Fourteenth Amendment.
Of the choices presented, the only likely basis to strike down the statute is under the Due Process Clause as a violation of substantive due process. Substantive due process tests the reasonableness of a statute; it prohibits arbitrary governmental action. Under substantive due process, when government action limits a fundamental right, the government must prove that the action is necessary to promote a compelling interest. If a fundamental right is not involved, the challenging party must prove that the act is not rationally related to any legitimate government interest. The retail sale of diapers is not a fundamental right, and so a challenger must prove that there is no rational basis for the statute. Almost any law can be justified under the rational basis standard. The law need not be the best law for accomplishing the government's goal. Thus, even if it is true that the disposable diaper ban causes more pollution than it prevents, because the ban is rationally related to reducing the volume of trash in landfills, the challenge is unlikely to succeed. Nevertheless, none of the other choices states a viable ground for invalidating the statute, and so (B) is the best choice. (A) is wrong because equal protection applies where a statute or governmental action treats similar people in a dissimilar manner (i.e., classifies people), and here there is no classification—under the statute no one can sell disposable diapers for use within the state. Thus, an equal protection argument is not applicable. (C) is wrong because the Impairment of Contracts Clause prohibits only the substantial impairment of existing contracts (and there are exceptions even where there is substantial impairment), and nothing in the facts indicates that forbidding the retail sale of disposable diapers would substantially impair any existing contract. (D) is wrong because the privileges and immunities covered by the Fourteenth Amendment are those attributes peculiar to United States citizenship (e.g., the right to petition Congress for redress or the right to vote for federal officers). The statute here does not affect such rights.
A blacksmith ran a small forge in a tourist attraction depicting village life in the 1800s, and produced small metal trinkets for sale as souvenirs. A tourist came into the forge and started ridiculing the blacksmith, telling him that he was foolish for practicing such an out-of-date trade when modern equipment could produce the same trinkets faster and far more cheaply. Although he maintained a calm demeanor, the blacksmith was enraged by the time the customer finished and headed back out the door. He picked up an anvil and hurled it in the general direction of the customer. The anvil fell harmlessly to the ground after traveling maybe a foot. If the blacksmith is charged with assault, which of the following statements would be most helpful for his defense? response - incorrect A The blacksmith did not succeed in hitting the customer with the anvil, and he knew that it was impossible to do so. B The blacksmith knew that it was impossible to hit the customer with the anvil. C The customer did not see the blacksmith throw the anvil, and the blacksmith knew that it was impossible to hit the customer with the anvil. D The customer did not see the blacksmith throw the anvil.
That the customer did not see the blacksmith throw the anvil, and that the blacksmith knew it was impossible to hit the customer with the anvil, would be most helpful to the blacksmith's defense. For purposes of the MBE, an assault is either (i) an attempt to commit a battery, or (ii) the intentional creation, other than by mere words, of a reasonable apprehension in the mind of the victim of imminent bodily harm. The fact that the blacksmith knew that it was impossible to hit the customer with the anvil negates the specific intent to commit a battery that is required for the first type of assault. (If the blacksmith knew that, when he threw the anvil, it was impossible to hit the customer, the blacksmith's conduct was not motivated by the intent to commit a battery against the customer.) The fact that the customer did not see the blacksmith throw the anvil negates the second type of assault because no apprehension of harm would have been created in the customer if he did not see the blacksmith throw the weight. Because the type of assault is not specified here, (C) is a better choice than (B) or (D) because both types of assaults are negated. Choice (A) is not correct because the fact that the blacksmith failed in his attempt to hit the customer with the anvil establishes only that there was a failure to commit a battery. It does nothing to negate the blacksmith's potential liability for assault/attempted battery.
A homeowner and a local builder entered into a written contract that called for the builder to build a second story onto the top of the homeowner's one-story residence. When scheduling conflicts arose, the builder asked the homeowner if they could substitute his buddy, an out-of-town builder who had comparable experience and skills, to perform the local builder's part of the contract. All of the parties agreed to the substitution. Unfortunately, the out-of-town builder made a major blunder that will be quite expensive to correct. Is the local builder liable to the homeowner for the cost of correcting the defect? response - incorrect A Yes, because the substitution in and of itself does not relieve the local builder of liability on the underlying contract. B Yes, because the local builder did not give any consideration on which to base a release. C No, because the local builder transferred his duties to the out-of-town builder. D No, because the local builder was discharged through a novation.
The agreement among all of the parties to substitute the out-of-town builder for the local builder operates as a novation which immediately discharged the local builder from any duties he had under the original contract. A novation arises when the parties enter into an agreement to substitute a third party for one of the parties in a contract, releasing the party who was substituted. All parties must agree to the substitution. Here, the facts say that all of the parties agreed that the out-of-town builder would substitute for the local builder. Thus, there was a novation and the local builder was released immediately and is not liable for the out-of-town builder's blunder. (A) states incorrectly that the substitution does not relieve the local builder of liability. If the parties had not all agreed to substitute the out-of-town builder for the local builder, or the facts said that there was merely an assignment of rights and delegation of duties, (A) would reflect the correct result. However, where the parties agree to substitute a new party for an old party, there is a novation that does release the old party. (B) is incorrect because there was consideration to support the release—the local builder implicitly agreed to give up his rights under the original contract, the homeowner implicitly agreed to give up his right to look to the local builder for performance, and the out-of-town builder agreed to perform. (C) is not as good an answer as (D). The mere fact that a contractual duty was transferred does not release the transferor from a duty under the contract. It is only the agreement among the parties to substitute the new party for the old that released the local builder here.
A town in a rural state facing financial difficulties passed a variety of "sin taxes," including one aimed at electronic game arcades frequented by local juveniles. The tax is a one cent per game tax imposed on the manufacturers of the games based on the estimated number of plays over a machine's lifetime. There are no electronic game manufacturers in the state. Which of the following constitutional provisions would support the best argument against enforcement of the tax? response - incorrect A The Equal Protection Clause. B Substantive due process. C The Privileges and Immunities Clause of Article IV. D The Commerce Clause.
The best argument against enforcement of the tax is that it violates the Commerce Clause. If Congress has not adopted laws regarding a subject, local governments are free to tax or regulate local aspects of the subject area as long as the tax or regulation does not discriminate against interstate commerce or unduly burden it. Here, the tax does not discriminate against interstate commerce, since it does not single out interstate commerce for taxation in order to benefit the local economy. However, it could be argued that the tax unduly burdens interstate commerce. A local tax will be held to unduly burden interstate commerce if the locality's need for the revenue does not outweigh the burden on interstate commerce. The Supreme Court will consider whether there is a substantial nexus between the activity or property taxed and the taxing state, whether the tax is fairly apportioned, and whether there is a fair relationship between the tax and the benefit the taxed party receives from the state. Here, there is little nexus between the manufacturer and the town. The facts indicate that out-of-state manufacturers' machines are used in the town, but do not indicate whether the manufacturers conduct any selling activity in the town. Similarly, nothing indicates that there is a relationship between the tax and any benefit that the manufacturers derive from the town. Thus, the tax would probably be unconstitutional under the Commerce Clause. (A) is not as good an argument as (D) because the Equal Protection Clause prohibits the states from treating similarly situated persons differently without sufficient justification. Where a classification does not involve a suspect or quasi-suspect class or a fundamental right, the classification will be upheld as long as it is rationally related to a legitimate government interest. While the tax here singles out arcade game manufacturers for special tax treatment, no suspect or quasi-suspect class is involved, nor is a fundamental right affected. Thus, the tax will be valid under the Equal Protection Clause because it is rationally related to the legitimate government interest of raising revenue. (B) is not a good argument because substantive due process requires that laws not be arbitrary. When laws do not involve a fundamental right, they will be held valid under the Due Process Clause as long as they are rationally related to a legitimate government interest. As established above, no fundamental right is involved and the tax is rationally related to a legitimate government interest. Thus, under the Due Process Clause the tax may be enforced. (C) is not a good argument because the Privileges and Immunities Clause of Article IV prohibits states from discriminating against out-of-state residents when a fundamental right is involved, and the tax here does not differentiate between residents and nonresidents.
A man who belonged to an ancient religion whose rituals require the use of bald eagle feathers traveled to an area where bald eagles were known to roost. After searching the area, he found a fallen eagle feather and returned home. A few weeks later, the man showed the feather to an acquaintance, who happened to be a state park ranger, and explained how the feather was obtained. The ranger informed the man that a state anti-poaching law makes any possession of a bald eagle feather without a special permit a crime. The ranger then cited the man for possession of the feather and confiscated it. At the man's trial for violating the state bald eagle feather possession statute, which of the following constitutional arguments is most appropriate for the prosecution to make? response - incorrect A The statute is a neutral law that only incidentally burdens the man's rights under the First Amendment. B The Free Exercise Clause applies only to belief and not to conduct. C The government has a substantial and important interest in protecting bald eagles and there is no other feasible way to achieve the legislative purpose. D Making an exception for the man on religious grounds would violate the Establishment Clause of the First Amendment.
The best argument for the prosecution is that the Free Exercise Clause does not afford a right to a religious exemption from a neutral law that happens to impose a substantial burden on a religious practice, if the law is otherwise constitutionally applied to persons who engage (or fail to engage) in the particular conduct for nonreligious reasons. Here, the state law interferes with the man's religious beliefs. However, the statute prohibits any possession of a bald eagle feather without a permit. Thus, the state should argue that the law was enacted to protect eagles and not merely to interfere with the religious beliefs of people such as the man here. (B) is incorrect because it is too broad. Conduct is protected (although the protection is limited). For example, the government cannot punish conduct merely because it is religious (although if the law affects both religious and nonreligious conduct, it is generally valid). [See, e.g., Employment Division v. Smith (1990)] (C) is incorrect because it states the former rule in these cases. In the past, the Court used a balancing test to determine whether a religious exemption had to be granted from a law with a secular purpose that happened to burden religious practices or beliefs. The Court would consider the severity of the burden, the strength of the state's interest, and the existence of alternative means. Now, however, the Court no longer uses a balancing test in actions involving state laws; the state need not establish a strong interest or a lack of alternative means if the challenged statute is neutral. (D) could be successfully argued, but its chances for success are not as certain as for the argument in (A). The Establishment Clause prohibits laws respecting the establishment of religion. If a law includes a preference for one religious sect over another, the law will be held invalid unless it is narrowly tailored to promote a compelling interest. If there is no sect preference, the law is valid if: (i) it has a secular purpose; (ii) its primary effect neither advances nor inhibits religion; and (iii) it does not produce excessive government entanglement. Here, no sect preference appears, because nothing indicates that an exception would apply only to members of the man's religion. It could be argued, however, that the only purpose for an exemption here is to favor religious believers over nonbelievers. If that is the purpose, the exemption would not have a secular purpose and would fail the secular purpose test above. On the other hand, the state could argue that free exercise of religion is also protected, and an exemption protects sincerely held religious beliefs. [See, e.g., Wisconsin v. Yoder (1979)] Thus, the outcome of the argument in (D) is uncertain, and (A) is the state's most appropriate argument.
The owner of a boat took two friends out on a lake near his home. One of his friends was driving the boat when it struck a partially submerged rock that the owner of the boat had forgotten to tell him about. The owner of the boat and the other passenger were injured; the driver of the boat was not hurt. In a jurisdiction that applies joint and several liability with comparative contribution, the passenger brought suit against both the boat owner and the driver, and the boat owner also sued the driver. The jury determined that the boat owner was 55% at fault and suffered $10,000 in damages, the driver of the boat was 45% at fault, and the injured passenger suffered $100,000 in damages. After entry of judgment, the boat owner paid the passenger her total damages of $100,000, while the driver of the boat has paid nothing. How much, if anything, can the boat owner recover from the driver? response - correct A $45,000, because the driver was 45% at fault. B $49,500, because the driver was 45% at fault and the boat owner suffered $10,000 in damages. C $50,000, because the boat owner and the driver are jointly liable. D Nothing, because the boat owner was more at fault than the driver.
The boat owner can recover $45,000 through comparative contribution for the passenger's claim and $4,500 on his own claim against the driver of the boat. Most comparative negligence states have adopted a comparative contribution system based on the relative fault of the various tortfeasors. Nonpaying tortfeasors who are jointly and severally liable are required to contribute only in proportion to their relative fault. Here, because the jurisdiction retained joint and several liability, the boat owner had to pay the passenger all of her damages. Under comparative contribution rules, the boat owner can obtain contribution from the driver for 45% of that amount, because the driver was 45% at fault. In addition, the boat owner has a direct claim against the driver for his own damages of $10,000, reduced by 55%, the amount of his fault. Thus, the total amount that the boat owner can recover from the driver is $49,500, making (B) correct and (A) incorrect. (C) is incorrect because it reflects traditional contribution rules, in which all tortfeasors were required to pay equal shares regardless of their respective degrees of fault. (D) is incorrect because a tortfeasor who was jointly and severally liable is not precluded from recovering contribution merely because he was more at fault than the other tortfeasors.
A landowner included in his will a provision giving "all of my property, both real and personal, wherever situated, to my widow for life, and after her death to any of our children who may survive her." What is the gift to the children? response - incorrect A A contingent remainder. B A vested remainder. C A shifting executory interest. D Void, as violating the Rule Against Perpetuities.
The children have a contingent remainder. A remainder is a future interest created in a transferee that is capable of taking in present possession on the natural termination of the preceding estate created in the same disposition. Note that, as a rule of thumb, remainders always follow life estates. A remainder will be classified as contingent if its taking is subject to a condition precedent, or it is created in favor of unborn or unascertained persons. Here, the interest in the children follows a life estate and is a remainder because it is capable of taking in possession on the natural termination of the preceding estate. It is subject to the condition precedent of surviving the landowner's widow and, additionally, is in favor of unascertained persons (the children who survive the landowner's widow will not be ascertained until her death). Thus, the interest is a contingent remainder. (B) is incorrect because a vested remainder can be created in and held only by ascertained persons in being, and cannot be subject to a condition precedent. As discussed above, the will provision clearly does not satisfy these requirements because the takers are not ascertained and their interest is subject to a condition of survival. (C) is incorrect because a shifting executory interest is one that divests the interest of another transferee; i.e., it cuts short a prior estate created by the same conveyance. The gift to the children does not divest the interest of the widow; she retains a life estate in the property. The children's interest takes in possession only on the natural termination of the widow's estate (i.e., at her death). (D) is incorrect because the interest does not violate the Rule Against Perpetuities. The children's interest will vest, if at all, not later than 21 years after the lives in being. The landowner's widow and the children themselves are lives in being. There is no unborn widow problem because the instrument takes effect on the landowner's death and the gift is to his own widow. She must be in being at his death. Likewise, his children would be in being at his death. Thus, the vesting will be within the period of the Rule.
A salvage company offered for sale Confederate dollars that had been recovered when the company recently raised a shipwreck off the coast of South Carolina. A purchasing agent for a private west coast museum purchased the bills, but he had represented that he was buying them for himself in hopes of obtaining a lower price. After purchasing the bills, the agent carefully packaged them and had them shipped to his museum. While the bills were in transit, the museum burned to the ground and its owner decided that she would not rebuild because most of her collections had been destroyed. When the bills arrived after the fire, the owner opened the package only to discover that the bills were too brittle for shipping by this method—three bills had disintegrated in transit. Undaunted, the owner took the remaining nine bills and had them mounted behind a glass frame so she could display them in her study. While the bills were being framed, the owner read on the Internet that a large cache of similar bills had just been discovered, and the market price for such bills had just been cut in half. Frustrated but still undaunted, the owner hung the framed bills in her study. Unfortunately, the salt water had reacted with the pigments in the bills in such a way that shortly after they had been exposed to indirect sunlight, all of the color in the bills faded almost completely away. No other Confederate bills raised from the ocean before had similar reactions; these bills appear to have been printed using substandard dyes. Which of the following facts would give the museum owner the best basis for rescinding the contract with the salvage company? response - incorrect A The bills were too brittle for transport. B The discovery of a large cache of similar bills a few days after the sale. C The bills' unusual reaction to indirect sunlight. D The destruction of the museum before the bills arrived.
The circumstances of (C) offer the best grounds for rescinding the contract based on mutual mistake. When both parties entering into a contract are mistaken about existing facts relating to the agreement, the contract may be voidable by the adversely affected party if (i) the mistake concerns a basic assumption on which the contract is made; (ii) the mistake has a material effect on the agreed-upon exchange; and (iii) the party seeking avoidance did not assume the risk of the mistake. Here, both parties probably believed that the bills would be suitable for display, like other bills that had been raised from the ocean. They had no reason to suspect that the bills would discolor when exposed to indirect sunlight. This occurrence probably rendered the bills nearly worthless, creating a material imbalance in the exchange. Finally, there is nothing to indicate that the museum owner/purchasing agent assumed the risk of what occurred. (A) is incorrect. Since the museum's agent was responsible for transporting the bills, the museum clearly assumed the risk that the bills were too brittle for the type of transport. (B) is incorrect because this is a fact occurring after the contract was made. When both parties entering into a contract are mistaken about existing facts (not future happenings) relating to the agreement, the contract may be voidable by the adversely affected party. Here, the subsequent change in price cannot be considered a mistake that was made at the time the parties entered into their contract. (D) is incorrect because the circumstances do not satisfy the requirements for discharge by frustration. Frustration will exist where the purpose of the contract has become valueless by virtue of some supervening event not the fault of the party seeking discharge. To establish frustration, the following must be shown: (i) there is some supervening act or event leading to the frustration; (ii) at the time of entering into the contract, the parties did not reasonably foresee the act or event occurring; (iii) the purpose of the contract has been completely or almost completely destroyed by this act or event; and (iv) the purpose of the contract was realized by both parties at the time of making the contract. Here, the salvage company thought that the purchasing agent was purchasing the bills for himself; thus, it did not realize at the time the contract was made that the purpose of the contract was to procure the bills for the museum that was subsequently destroyed. Therefore, frustration will not be available as a ground for rescission here.
The defendant and an accomplice were on trial together for burglary. Both had given confessions implicating themselves and their accomplice. At trial, the defendant maintained that his confession had been obtained through improper coercion by the police. For the purpose of countering the claim of coercion, the prosecution seeks to place the accomplice's confession into evidence. After objection by the defendant's counsel, the judge agrees to issue a limiting instruction to the jury that the confession is to be considered only with regard to the question of whether the defendant's confession was coerced. May the accomplice's confession be admitted under that condition? response - incorrect A No, because admission of the confession violates the defendant's right of confrontation. B No, unless the accomplice takes the stand and subjects himself to cross-examination regarding the confession. C Yes, as long as all portions of the confession referring to the defendant can be eliminated. D Yes, because the judge's instruction limits consideration of the confession only to the issue of coercion.
The confession is admissible with the judge's limiting instruction. Where two persons are tried together and one has given a confession implicating the other, the general rule is that the Sixth Amendment right to confront adverse witnesses prohibits the use of such a statement. This problem arises because of the inability of the nonconfessing defendant to compel the confessing co-defendant to take the stand for cross-examination at their joint trial. As exceptions to the general rule, the statement may be admitted if: (i) all portions of the statement referring to the other defendant can be eliminated (so that there is no indication of that defendant's involvement); (ii) the confessing defendant takes the stand and subjects himself to cross-examination with respect to the truth or falsity of what the statement asserts; or (iii) the confession of the nontestifying co-defendant is being used to rebut the defendant's claim that his confession was obtained coercively, in which case the jury must be instructed as to the purpose of the admission. The accomplice's confession, which the prosecution seeks to introduce into evidence, implicates the defendant in the commission of the crimes charged. Consequently, introduction of this confession raises a problem based on the right of confrontation. However, given that the judge will issue the limiting instruction, the confession is admissible. (D) is therefore correct and (A) is incorrect. (B) and (C) are incorrect because neither of those conditions is necessary for the confession to be admitted as long as the judge issues a limiting instruction, as discussed above.
A man and a woman were arrested and charged with a series of armed robberies. Each suspect was given Miranda warnings, and different interrogation teams questioned each suspect separately. Upon being questioned, the man told the police, "I'm not going to talk until I see a lawyer." An officer responded, "You might want to reconsider, because your partner has already confessed, and she's implicated you in the crimes." The man then told the police that he wanted to talk to the woman privately. The police escorted the man to the woman's cell, locked him in with her, and left. Unbeknownst to either of them, the police had bugged the woman's cell and recorded both the man and the woman making self-incriminating statements during their meeting. The man made no further statements to the police on advice of counsel, whom he called immediately after his conversation with the woman. The man was put on trial first, and the prosecution sought to introduce into evidence tapes of the bugged conversation between the man and the woman. The defense made a motion to suppress the evidence. Should the court grant the motion to suppress?
The conversation should be suppressed because the police conduct violated the man's Sixth Amendment right to counsel. The Sixth Amendment provides that in all criminal prosecutions a defendant has a right to the assistance of counsel at all critical stages after formal proceedings have begun. For Sixth Amendment purposes, a criminal prosecution begins when adversary judicial proceedings have commenced, such as the filing of formal charges in this case. Because custodial interrogation is a critical stage of prosecution, the Sixth Amendment is violated by post-charge interrogation unless the defendant has waived his right to counsel. Interrogation includes not only direct questioning, but also any other conduct by the police intended to elicit a response. The police conduct here (telling the man that the woman had implicated him and then bugging the conversation) constitutes prohibited interrogation. [See Maine v. Moulton (1985)] (A) is incorrect because the wiretap was not an illegal search under the Fourth Amendment. Wiretapping and other forms of electronic surveillance are subject to the Fourth Amendment prohibition of unreasonable searches and seizures. However, to have a Fourth Amendment right, a person must have a reasonable expectation of privacy with respect to the place searched or the item seized. In a different context, the Supreme Court has held that prisoners have no reasonable expectation of privacy in their cells or in any personal property that they have in their cells. [Hudson v. Palmer (1984)] Hence, neither defendant can assert a Fourth Amendment claim based on the wiretap, because they had no reasonable expectation of privacy in the jail cell. The fact that there was no expectation of privacy does not make choice (C) correct, however. Even though he probably cannot claim that the bugging was an unreasonable search under the Fourth Amendment, the man can claim that it was an interrogation in violation of his Sixth Amendment right to counsel, as discussed above. (D) is incorrect because it is irrelevant. The facts probably would not give rise to a Miranda violation in light of the Court's ruling in Illinois v. Perkins (1990) that Miranda does not apply unless interrogation is by someone known to be a police officer (on the rationale that Miranda is merely a prophylactic rule designed to offset the coercive nature of a custodial interrogation by a police officer). In any case, Miranda rights and Sixth Amendment rights to counsel can only be waived knowingly, and so the man's ignorance of the fact that the cell was bugged precludes a finding of waiver here.
A plaintiff filed a civil action based on negligence against a defendant in federal district court, alleging that the defendant negligently ran a red light at an intersection and collided with the plaintiff's vehicle, causing the plaintiff's injuries. A week after the close of discovery, the plaintiff filed a motion for summary judgment on the issue of whether the defendant was negligent. With the motion, the plaintiff filed (i) his own sworn affidavit, which stated that the traffic signal was green as he entered the intersection; (ii) an affidavit of a witness who was driving the car behind him, which stated that the witness saw the entire incident and that the plaintiff's traffic signal was green as he approached and entered the intersection; and (iii) an affidavit of another witness, which stated that she saw the entire incident and that the defendant's signal had been red for several seconds before the defendant entered the intersection and was still red when the defendant entered the intersection. In response to the motion, the defendant filed his own affidavit which stated that he does not recall seeing the traffic signal before entering the intersection, but believes that it was not red. He also filed the affidavit of a pedestrian on the scene. The pedestrian stated that she did not see the traffic signal prior to the accident, but that another passerby told her that the traffic signal was yellow for both drivers. How should the court rule on the plaintiff's motion? response - incorrect A Grant the motion, because the plaintiff has supported his motion with substantial evidence and the defendant has failed to produce admissible evidence that contradicts the plaintiff's evidence. B Grant the motion, because the defendant's conduct is still negligent even if the defendant's traffic signal was yellow. C Deny the motion, because the defendant has presented evidence that contradicts the evidence presented by the plaintiff. D Deny the motion, because a party may not obtain summary judgment on an issue on which that party has the burden of proof.
The court should grant the motion. Summary judgment may be granted if, from the pleadings, affidavits, and discovery materials, it appears that there is no genuine dispute of material fact and the moving party is entitled to judgment as a matter of law. The court may not decide disputed fact issues on a motion for summary judgment; if there is a genuinely disputed material fact (meaning a dispute backed by evidence on both sides of the issue), the case must go to trial. Here, the plaintiff's evidence that the light was green when he entered the intersection has not been contradicted by admissible evidence. The defendant's affidavit essentially does not dispute the color of the light, as the defendant cannot remember what color it was. The pedestrian's testimony constitutes inadmissible hearsay. Thus, there is no material fact in dispute, and the motion should be granted. (Alternatively, the court could delay in ruling on the motion to give the defendant an opportunity to find the unavailable witness who spoke with the pedestrian. However, that is not an answer choice, and it is unclear from the facts whether that witness could even be found.) (B) is not correct, as it would become a matter for the jury to decide if running a yellow light is negligent. (C) is incorrect because the defendant has not produced admissible evidence that contradicts the plaintiff's position. (D) is an incorrect statement of the law. Summary judgment may be granted on an issue as to which the party has the burden of proof.
A patient properly sued her doctor in federal court for medical malpractice. At trial, the doctor's attorney called four well-known experts in the field who uniformly agreed that the doctor acted within the normal standard of care in treating the patient. The patient's attorney called only one young and inexperienced expert who opined that the doctor did not act within the normal standard of care; however, he contradicted himself on the stand and could not answer certain simple questions. During the trial, no motions were made by the doctor's attorney. Surprisingly, the jury returned a verdict in favor of the patient, and the doctor's attorney filed a renewed motion for judgment as a matter of law 22 days after the judgment was entered. Will the court grant the motion? response - incorrect A Yes, because the judgment was based on a verdict that a reasonable jury would not have had a legally sufficient basis to reach. B Yes, because there was no genuine dispute of material fact and the moving party was entitled to judgment as a matter of law. C No, because the doctor did not move for a judgment as a matter of law during the trial. D No, because a renewed motion for judgment as a matter of law must be filed within 21 days after the judgment is entered.
The court should not grant the motion. A judgment notwithstanding the verdict ("JNOV") is now called a renewed motion for a judgment as a matter of law. To be valid, the party making the renewed motion must have moved for judgment as a matter of law at some time during the trial. Here, the doctor never moved for a judgment as a matter of law during the trial. Therefore, his motion for a renewed judgment as a matter of law will be denied. (A) is wrong. Although it states the correct standard of review for a renewed motion for judgment as a matter of law (the judgment was based upon a verdict that a reasonable jury would not have had a legally sufficient basis to reach a verdict), the problem in this question is that the doctor failed to move for a judgment as a matter of law at trial. Therefore, this motion cannot even be considered. (B) is wrong because this is the standard used to review a motion for summary judgment. (D) is wrong because it is a misstatement of law. A renewed motion for judgment as a matter of law must be filed no later than 28 days after the judgment is entered, not 21 days.
A woman sued her ex-husband for installments due under their divorce property settlement agreement. The ex-husband defended the suit on the ground that the ex-wife was in breach of the agreement. However, the court granted judgment for the ex-wife. Now, the ex-wife is suing her ex-husband for other installments due under the agreement. The ex-husband raises the defense that the agreement was void and illegal because of fraud perpetrated by his ex-wife at the time the agreement was signed. The ex-wife moves to strike her ex-husband's defense, claiming issue preclusion. Should the court grant the motion? response - incorrect A No, because the issue was never litigated. B No, because judgment on one installment does not bar subsequent actions on other installments. C Yes, because the husband should have raised the defense in the first action. D Yes, because the issue of fraud is never waived.
The court should not grant the motion. Generally, issues actually litigated between the parties are binding on them in subsequent actions concerning the same claim. If the second suit involves a different claim, the first judgment may be invoked as to all matters actually litigated and determined in the first action, provided that the findings were essential to the first judgment. Here, the fraud issue was not actually litigated in the first action, and thus it can be raised with respect to the later installments. (B) is incorrect. The general principle recited in this answer is true for claim preclusion (unless there is an acceleration clause). However, even if claim preclusion does not apply, if the defendant raises the same issues as in the first case, he may be prevented by issue preclusion (collateral estoppel) from re-litigating those issues in that subsequent case. (C) is incorrect. Claim preclusion (res judicata) would bar claims arising out of the same transaction or occurrence. Here, claim preclusion does not apply because a defense, not a claim, is involved, and the second installment is a separate transaction or occurrence. (D) is incorrect because there is no such rule.
A defendant visited her doctor to seek treatment for a bullet wound. While he was treating the wound, the doctor asked the defendant how she was shot. The defendant replied that she was struck by a police officer's bullet while running away from a jewelry store she had robbed, but she implored the doctor not to tell this to anyone. The doctor promised that he would not. Although the defendant was never charged by the police, the owner of the jewelry store brought suit against her seeking the value of the stolen goods. The defendant denied robbing the store. At the trial, the owner calls the doctor to testify to the statement made to him by the defendant. The defense attorney objects on the ground that such testimony is barred by the jurisdiction's physician-patient privilege. Should the objection be sustained? response - incorrect A Yes, because the doctor acquired this information while attending the defendant in the course of treatment. B Yes, because the doctor agreed to the defendant's specific request that this information be kept confidential. C No, because the physician-patient privilege is inapplicable to the defendant's statement. D No, because the doctor is the one who is entitled to either claim this privilege or waive it.
The court should overrule the objection because the physician-patient privilege cannot be invoked for information dealing with a nonmedical matter. Under the physician-patient privilege, a physician is foreclosed from divulging in judicial proceedings information that he acquired while attending a patient in a professional capacity, which information was necessary to enable the physician to act in his professional capacity. Information given by a patient that deals with a nonmedical matter is not protected by the privilege. Hence, the defendant's admission that she was shot while running from a jewelry store that she robbed is not barred by the privilege. (A) is incorrect because, although it is true that the doctor acquired the information while attending the defendant in the course of treatment, the privilege is inapplicable because, as discussed above, the statement deals with a nonmedical matter. (B) is incorrect because a promise to comply with a request by the patient that information be kept confidential will not by itself render the information protectable under the physician-patient privilege. To qualify for such protection, the information must have been necessary for treatment, and there must be no applicable exceptions to the privilege. (D) is incorrect because this privilege belongs to the patient. Thus, the defendant is the one who is entitled to claim or waive the privilege, not the doctor.
The criminal statutes of the state define manslaughter and murder as they were defined at common law. As to insanity, the state has the following provision: "Under the defense of insanity a defendant may be entitled to acquittal if, because of mental illness, the defendant was unable to control his or her actions or to conform his or her conduct to the law." The defendant was put on trial in the state for the murder of his wife and her co-worker. The evidence at trial established that the defendant's wife was having an affair with the co-worker, and that the defendant learned of it and killed the pair. The defendant did not take the stand in his own defense. In his closing statement to the jury, the defendant's attorney made a statement, "Ladies and gentlemen, you must consider that there are some things that would provoke any one of us to kill, and there are things that make one unable to control one's actions." The defendant's attorney requested that the judge give the jury instructions on manslaughter and on insanity, and the judge agreed to do so. The judge also issued the following instructions: "INSTRUCTION #6: In order to mitigate an intentional killing to voluntary manslaughter, the burden of proof is on the defendant to establish that adequate provocation existed." "INSTRUCTION #8: Insanity is an affirmative defense and the burden of proof is on the defendant to establish that such insanity existed at the time of the killing." The jury found the defendant guilty of murder, and he appealed. He asserts that the jury instructions violated his rights under the federal Constitution. How should the appeals court rule? response - incorrect A Reverse the defendant's conviction, because Instruction #6 was improper. B Reverse the defendant's conviction, because Instruction #8 was improper. C Reverse the defendant's conviction, because both Instructions #6 and #8 were improper. D Uphold the defendant's conviction, because neither Instruction #6 nor Instruction #8 was improper.
The court should reverse the defendant's conviction because Instruction #6 requires the defendant to disprove one of the elements of murder. Due process requires in criminal cases that the state prove guilt beyond a reasonable doubt. The prosecution has the burden of proving all of the elements of the crime charged. Thus, if malice aforethought is an element of murder and voluntary manslaughter is distinguished from murder by the existence of adequate provocation, the defendant cannot be required to prove that he committed the homicide in the heat of passion (i.e., with adequate provocation). Such a requirement would impose on the defendant the burden of disproving the element of malice aforethought, because "heat of passion" negates malice. Although the defendant can be given the burden of going forward with some evidence on the provocation issue, once he has done so, the prosecution bears the burden of proving that the killing was not done in the heat of passion. In the case at issue, Instruction #6 requires a defendant to prove that he committed the intentional killing under adequate provocation. At common law, and consequently in the state, malice aforethought is an element of murder. Therefore, this instruction in effect requires the defendant to disprove the element of malice aforethought, thereby relieving the state of its burden of proving all elements of the crime. As discussed above, such an instruction cannot pass constitutional muster. On the other hand, for an affirmative defense such as insanity, it is permissible to impose the burden of proof on the defendant. Thus, Instruction #8 does not affect the state's obligation to prove all elements of the crime, and is permissible under the general principles mentioned above. Thus, (B) and (C) incorrectly state that this instruction is improper. (D) is incorrect because it states that Instruction #6 is proper. As explained above, this is not an accurate statement of the law.
A landlord leased office space to a business owner for five years, ending on November 1, reserving a yearly rent of $24,000, payable monthly. On October 1 of the fifth year, the business owner notified the landlord that he was preparing to move, but would greatly appreciate if the landlord could extend the lease for a month or two. On October 10, the landlord wrote to the business owner that she thought they could reach a satisfactory arrangement, but did not hear back from the business owner. The business owner did not vacate the office until November 20. On November 30, the landlord received a check from the business owner in the amount of $1,333 for "November's rent" and a note that he had vacated the premises. If the landlord brings an action against the business owner for additional rent, how will the court rule? response - incorrect A The business owner is bound to a year-to-year tenancy, because he did not vacate the premises until November 20. B The business owner is bound to a tenancy through December because one month's advance notice was required to terminate. C The business owner is not bound, because the $1,333 check discharged him from his obligations. D The business owner is not bound if the court admits parol evidence of the October 10 letter from the landlord.
The court will rule that the business owner is bound to a year-to-year tenancy because he is a hold-over tenant. When a tenant fails to vacate the premises after the termination of his right to possession, the landlord may: (i) treat the hold-over tenant as a trespasser and evict him; or (ii) bind the tenant to a new periodic tenancy. The terms and conditions of the expired tenancy apply to the new tenancy. At least in commercial leases, the new tenancy will be year-to-year if the original lease term was for one year or more. Here, the businessman was a tenant for years because his lease was for a five-year fixed period of time. A tenancy for years ends automatically on its termination date. Therefore, as of November 1, the business owner became a hold-over tenant and the landlord had a right to bind him to a new periodic tenancy. Because the original lease was for more than one year, the business owner may be held to a year-to-year tenancy, at the stipulated rent of $24,000 per year. (B) is incorrect because even though the rent is payable monthly, the majority view is that reservation of an annual rent results in a year-to-year periodic tenancy. Hence, his notice of termination on November 30 would not take effect until the end of the new tenancy. (C) is incorrect because the business owner's mere continuance in possession after November 1 gave the landlord the right to bind him to another year's term. This right was not affected by the fact that the business owner paid 20 days' worth of rent. Moreover, although a tenancy for years may terminate on surrender, surrender requires the landlord's acceptance, which is not evident here. (D) is incorrect because even if the court admits the October 10 letter, it merely indicates the landlord's willingness to consider an extension. Because the business owner did not respond and no agreement was reached by the parties, the letter is not enough to allow the business owner to avoid the additional tenancy.
The state has the following homicide statutes: Murder is the unlawful killing of a human being with malice aforethought. Such malice may be express or implied. It is express when there is manifested a deliberate intention to unlawfully take away the life of a fellow creature. It is implied when no considerable provocation appears or when the circumstances attending the killing show an abandoned and malignant heart. All murder that is perpetrated by willful, deliberate, or premeditated killing or committed in the perpetration of or attempt to perpetrate arson, rape, robbery, or burglary is murder of the first degree. All other kinds of murders are of the second degree. The defendant and her associate entered a jewelry store to shoplift a diamond bracelet. Just as the defendant put the bracelet into her pocket, a sales clerk saw her and grabbed her by the wrist. The associate grabbed a knife from one of the silver displays and lunged at the sales clerk, but then a store guard shot and killed her. The defendant is charged with the first degree murder of her associate. Which of the following is the defendant's strongest argument? response - incorrect A The defendant cannot be convicted of murder because when they went into the store they were not carrying any weapons; therefore, there was no felony on which the felony murder rule may arise. B The defendant cannot be convicted of murder because the associate's death was not murder but justifiable homicide. C The defendant cannot be convicted of murder because she and her associate had an agreement never to use violence when they stole anything. D The associate did not intend to hurt the sales clerk, but just wanted to scare him so that the defendant could run.
The defendant's strongest argument is that her associate's death was justifiable homicide. Most courts today would not allow the defendant to be convicted on a felony murder theory when a co-felon is killed by a third party during the crime. Some courts base this result on the fact that the person who did the killing was justified in doing so. (A) is wrong. Aside from the fact that the attempt to steal from the jewelry store is probably statutory burglary, the fact that the associate attempted to aid the defendant in stealing the bracelet by attacking the clerk with a knife is probably robbery. When or how the associate came by the dangerous weapon is immaterial. (C) is wrong because the circumstances of one co-felon breaking an agreement not to commit violence would not prevent the application of the felony murder rule if it were otherwise applicable. (D) is wrong because if the felony murder rule is otherwise applicable, the fact that the person who killed the co-felon may have mistaken the co-felon's intentions does not prevent the operation of this rule.
A plaintiff sued a defendant for defamation, asserting in her complaint that the defendant had called the plaintiff a thief in front of a number of business associates. The plaintiff calls two witnesses to the stand, both of whom testify that they heard the defendant refer to the plaintiff as a thief in front of the business associates. The plaintiff does not take the stand herself. The defendant pleads truth of the statement as an affirmative defense and calls a witness to the stand. The defense witness is prepared to testify that he was a co-worker of the plaintiff when the plaintiff supplemented her income by tending bar three nights a week. The witness will testify that he saw the plaintiff take a $20 bill from the tavern's cash register and secrete the money in her pocket. The plaintiff's attorney objects. May the defense witness's testimony be allowed? response - incorrect A Yes, as substantive evidence that the plaintiff is, in fact, a thief. B Yes, because theft is a crime indicating dishonesty. C No, because specific bad acts may not be used to show bad character. D No, because the plaintiff never took the stand.
The defense witness's testimony is admissible character evidence because the plaintiff's character is directly in issue in the case. As a general rule, evidence of character to prove the conduct of a person in the litigated event is not admissible in a civil case. However, when proof of a person's character, as a matter of substantive law, is an essential element of a claim or defense in a civil action, character evidence is admissible because it is the best method of proving the issue. Under the Federal Rules, any of the types of evidence—reputation, opinion, or specific acts—may be used. Here, character is an issue in the plaintiff's defamation action because the defendant has pleaded as an affirmative defense that his statement claiming that the plaintiff is a thief is the truth. The defense witness's testimony that he saw the plaintiff take the money from the cash register is relevant because it tends to show that the defendant spoke the truth. Hence, it should be allowed. (B) is incorrect because the fact that the theft here could be considered a crime of dishonesty would be relevant only if the plaintiff's credibility were being impeached, and only then if proof of an actual conviction were provided. Here, the testimony is admissible because it is being offered as substantive evidence of an aspect of the plaintiff's character that is an essential element of a defense in the case. (C) is incorrect. One of the few cases where testimony as to specific acts of a person may be used to show that person's character is when character itself is one of the essential issues in the case, as it is here. (D) is incorrect because the fact that the plaintiff never took the stand only means that she has not placed her credibility in issue and become subject to impeachment. Here, however, the plaintiff's character is in issue and the testimony is being offered as substantive evidence of her character rather than to impeach her credibility.
While driving in a city in State A, a citizen of State B struck a pedestrian who was a citizen of State C. The pedestrian sued both the driver and the State A city in federal district court, seeking $100,000. The pedestrian alleged that the driver was liable for negligently operating his car and that the State A city was liable for negligently maintaining a traffic signal. The driver, who owns an office supply wholesaler, also has a contract claim against the State A city for $80,000 worth of furniture that the city allegedly purchased and received but never paid for. Assume that State A has waived any applicable governmental immunity. If the driver files a cross-claim for negligence against the State A city to recover damages for his injuries in the accident with the pedestrian, can the driver join with that cross-claim his contract claim for the purchase price of the furniture and maintain the claim in the same federal action? response - incorrect A No, because defendants cannot add claims against co-defendants that are not related to the original claims asserted by the plaintiff. B No, because the court does not have supplemental jurisdiction over the driver's contract claim against the State A city. C Yes, the driver must assert the contract claim or he will be foreclosed from asserting it later. D Yes, the driver may join his contract claim with his negligence cross-claim against the State A city, but is not required to do so.
The driver can join his contract claim with his negligence cross-claim, but he is not required to do so. As a general rule, a party may assert a cross-claim against a co-party only if the cross-claim arises from the same transaction or occurrence as that of the original action or of a counterclaim. However, once the party has filed such a cross-claim, he also may join with it any other claim that he has against the same party. Here, the driver's contract claim is unrelated to the pedestrian's negligence claim; however, the driver's negligence claim is related to the pedestrian's negligence claim. Thus, the contract claim also can be asserted in this case. (A) is therefore incorrect. (B) is incorrect because the court would have diversity jurisdiction over the contract case. (C) is incorrect because the contract claim is not a compulsory counterclaim.
A vintner divided his vineyard into two parcels, drawing the boundaries so that the single well that had irrigated the entire vineyard fell on the border of the two properties. The vintner then conveyed the eastern parcel to his friend by a deed that contained the following covenant: If the well located on the boundary of the eastern and western parcels continues to be used for irrigation purposes and becomes in need of repair or replacement, the grantee, his heirs, and assigns and the grantor, his heirs, and assigns each promise to pay one-half of the cost of such repair or replacement. This covenant shall run with the land. The deed from the vintner to the friend was not recorded, and the vintner did not record a copy of the deed with the records for the western parcel. The friend later sold the eastern parcel to a farmer. The farmer's deed did not contain the covenant about the well. After 15 years of use by the owners of both the eastern and western parcels, the well began to fail. The farmer took it upon himself to have the well repaired at a cost of $30,000. About two weeks later, the farmer discovered the deed from the vintner to the friend in some old files. By this time, the western parcel had passed to the vintner's son by inheritance and again to the son's daughter by inheritance from the now-deceased son. The daughter knew nothing of the covenant concerning the well. The farmer presented the daughter with the bill for the well repair with a copy of the vintner/friend deed and a note that said he expected to be reimbursed for $15,000. The daughter refuses to pay, and the farmer sues. The jurisdiction has a 10-year statute of limitations for acquiring property by adverse possession, and the following recording statute: "Any conveyance of an interest in land shall not be valid against any subsequent purchaser for value, without notice thereof, unless the conveyance is recorded." For whom is the court most likely to rule?
The farmer will most likely prevail in his suit for one-half the cost of the well repairs because the covenant runs with the land. When a covenant runs with the land, subsequent owners of the land may enforce or be burdened by the covenant. If all of the requirements for the burden to run are met, the successor in interest to the burdened estate will be bound by the arrangement as effectively as if he had himself expressly agreed to be bound. To be bound: (i) the parties must have intended that the covenant run with the land; (ii) the original parties must have been in horizontal privity; (iii) the succeeding party must be in vertical privity with the original promisor; (iv) the covenant must touch and concern the land; and (v) generally, the burdened party must have actual or constructive notice of the covenant. Here, the intent is shown by the express language of the covenant, which says that it is intended to run with the land. Even without that language, the use of the words "heirs" and "assigns" would show the intent for the covenant to run. The original parties were in horizontal privity because at the time the vintner entered into the covenant, he and the friend shared an interest in the land independent of the covenant—as grantor and grantee. The daughter is in vertical privity with the vintner because she holds the entire interest in the western parcel held by the vintner. The covenant touches and concerns the land because promises to pay money to be used in a way connected with the land are held to touch and concern the property. Because the daughter was unaware of the covenant, the required notice seems to be missing. While it is generally true that the owner of the burdened land must have notice, it should be remembered that the requirement is a function of the recording statute. (At common law, the covenant was enforceable in an action for damages regardless of notice; this was changed by the recording statutes.) However, because the daughter is a donee (an heir) and not a bona fide purchaser, she is not protected by the recording statute and thus is subject to the covenant even without notice. For that reason, (A) is wrong. (B) is wrong because the farmer's possession does not satisfy several of the requirements for adverse possession. Because the farmer had a legal right to use the well, his use was not adverse or hostile to the rights of the vintner's son and the son's daughter, but was rather permissive. The farmer's possession also fails the exclusivity requirement because the facts state that the well was used to irrigate both parcels for most of the statutory period. (D) is wrong because the farmer's status as a bona fide purchaser has no effect on his ability to enforce the covenant. A successor in interest to the original promisee may enforce the covenant (enjoy the benefit) if there was intent and vertical privity, and the covenant touches and concerns the land. Notice is not required for the benefit to run. Thus, because the above requirements are met here, the farmer may enforce the covenant regardless of his status as a bona fide purchaser. Had the farmer taken the property as a donee, the above analysis would be the same.
A father was angry at his son's coach because the coach would never let the son into a game. In order to exact revenge, the father decided to plant an incendiary device on the coach's front porch. The father believed the device would start a fire that would destroy the coach's home and perhaps injure him as well. However, the father made a mistake while assembling the incendiary device, and it was impossible for the device to do any harm. When the device went off, it did nothing more than produce a foul odor. If the father is charged with attempted murder and attempted arson in a common law jurisdiction, which of the following decisions is most likely to be reached by the court? response - incorrect A The father is guilty of attempted murder and attempted arson. B The father is guilty of attempted murder, but he is not guilty of attempted arson. C The father is not guilty of attempted murder, but he is guilty of attempted arson. D The father is not guilty of attempted murder or attempted arson.
The father lacked the specific intent to kill that is required for attempted murder. However, the circumstances surrounding the "incendiary device" constitute factual impossibility and will not afford the father a defense to attempted arson. Criminal attempt is an act that, although done with the intention of committing a crime, falls short of completing that crime. To be guilty of attempt, the defendant must have the intent to perform an act and obtain a result that, if achieved, would constitute a crime. Regardless of the intent that would suffice for the completed offense, attempt always requires a specific intent to commit the target offense. Also, the defendant must have committed an act beyond mere preparation for the offense. Here, to be guilty of attempted murder, the father must have had the specific intent to kill his son's coach, even though the intent to inflict great bodily injury would be sufficient mens rea for murder. However, the facts indicate that the father intended at most only to injure the coach rather than kill him. Thus, the father cannot be guilty of attempted murder. However, the father did intend to burn the coach's home; therefore, he had the specific intent to commit arson by means of placing an incendiary device on the coach's porch, and his placing the device was an act beyond mere preparation for this crime. Although the device could not have actually burned the coach's house, it is no defense to attempt that it would have been impossible for the defendant to complete his plan. This is factual impossibility and is not a defense. Thus, the father is guilty of attempted arson. (A) and (B) are incorrect because the father did not have the specific intent to kill. (D) is incorrect because the father is guilty of attempted arson, as explained above.
A federal statute just signed into law by the President provided that school districts no longer needed to recognize the tenure of elementary school teachers—all tenured teachers would lose their status and would be treated the same as nontenured teachers. The effect of the law would be to allow all tenured teachers to be fired more easily if their performance was not adequate. The law also allowed the salaries of tenured teachers to be lowered, at least until a new contract with the teachers could be negotiated. The law had a two-year grace period before it was to take effect, to give schools and teachers time to adjust to the law; however, it specifically provided that once it is in effect, school board actions under the law supersede any existing contract terms. A public elementary school district is in the first year of a three-year union contract with its teachers. The school board has stated that it plans to abolish tenured positions as soon as the law takes effect. The union, believing that numerous terms of the contract will be invalidated when the law takes effect, filed an action in federal court on behalf of the teachers, asking for an injunction to prevent the school board from abolishing tenured positions and for a declaratory judgment stating that the law is invalid. Should the federal court hear the case? response - incorrect A No, because a ruling on the law at this point is premature. B No, because the union does not have standing to sue on behalf of the teachers. C Yes, because the federal law encourages improper interference with a contract in violation of the Contract Clause of the Constitution. D Yes, because the teachers' rights and benefits are threatened by the law and the school board's stated plans.
The federal court should not hear the case because it is not yet ripe for review. A federal court will not hear a case unless there exists a "case and controversy." This has been interpreted to mean, among other things, that a plaintiff generally is not entitled to review of his claim unless he has been harmed or there is an immediate threat of harm. This is to prevent the federal courts from hearing unnecessary actions. There is no immediate threat of harm to the union here because the law does not take effect for another two years. Before that happens, Congress might change the law or repeal it altogether, or the school board may decide to keep the old contract system after all. Thus, (A) is correct and (D) is incorrect. (B) is incorrect because the union would have standing. An association has standing if (i) there is an injury in fact to its members that would give them standing, (ii) the injury is related to the organization's purpose, and (iii) neither the nature of the claim nor the relief requested requires participation of the individual members in the lawsuit. All three of the conditions are met here; thus, the union would have standing. (C) is incorrect because the Contract Clause does not limit federal power, only state power, and because the state would be acting pursuant to a federal law here, there would be no Contract Clause violation. Moreover, even if the Contract Clause limited the state's actions here, it still is not clear that there would be a constitutional violation. The Clause bans only substantial interference with existing contracts (i.e., destruction of almost all of a party's rights under a contract), and it is not clear here that the impairments would be sufficiently substantial.
A mother's will left her farm to her son and daughter "jointly, as tenants in common." The son and the daughter, having had no interest in farming, had long since moved to a large city about 150 miles from the farm. However, after the mother's death the son decided to move back to the farm. The son rented various parts of the farm to sharecroppers and regularly sent half of any profits from the farm to the daughter. A few years later the daughter died, leaving a will devising all of her property to a friend. The son, however, refuses to pay any of the profits of the farm to the friend and claims an exclusive interest in the farm. If the friend sues the son, how will a court most likely rule? response - incorrect A For the son, because he actively managed the use of the farm and the daughter never showed any interest in it. B For the son, because he survived the daughter, the other joint tenant. C For the son, because the unities of time, title, and interest have been destroyed by the daughter's death. D For the friend, because he inherited the daughter's interest.
The friend will prevail because he inherited the daughter's interest. Although the language in the mother's will uses the word "jointly," the grant also states "as tenants in common." Because no right of survivorship is mentioned, the court will most likely find that this language establishes a tenancy in common, rather than a joint tenancy. The daughter can pass her interest in the farm by will, and thus the friend now holds the farm as a tenant in common with the son. (A) is wrong because the son's management of the use of the farm does not entitle him to an exclusive interest in it. (B) is wrong because the interest created by the mother's will was a tenancy in common, not a joint tenancy. (C) is wrong because the unities only apply to a joint tenancy.
A recent law school graduate was offered a job as an aide by a state legislator. The legislator told the graduate that before she could begin working, she had to take the following loyalty oath: "I swear to uphold our state and federal Constitutions; to show respect for the state and federal flags; and to oppose the overthrow of the government by violent, illegal, or unconstitutional means." The graduate told the legislator that the oath is unconstitutional and refused to take the oath. Is the graduate correct? response - incorrect A Yes, as to the promise to uphold the state and federal Constitutions. B Yes, as to the promise to respect the flag. C Yes, as to the promise to oppose the overthrow of the government. D No, as to all three provisions.
The graduate is correct as to the promise to respect the flag, but incorrect as to the other two promises. The Supreme Court has upheld oaths requiring government employees to oppose the violent overthrow of the government and to support the Constitution; hence, (A) and (C) are incorrect. However, it has held that government employees cannot be required to show respect for the flag, as a person might refuse to salute the flag on religious grounds. Thus, (B) is correct and (D) is incorrect.
A husband was on his way to meet his wife for lunch at the restaurant in the lobby of a bank building where she worked. He had just entered the building, which was owned and operated by the bank, when he heard screams and the sound of breaking glass from the restaurant area. He immediately saw that a large piece of artwork made of stained glass had fallen onto the seating area of the restaurant. In the seating area he saw several injured persons, including his wife, lying in the wreckage of the artwork. He fainted and hit his head on the marble floor, fracturing his skull. The artwork had collapsed because the pedestal that the bank had provided for the artwork was not properly constructed. If the husband sues the bank for his injury, is he likely to prevail? response - incorrect A No, because he was not personally in the zone of danger of physical injury. B No, because he did not actually see the artwork collapse onto the diners. C Yes, because his wife was one of the persons he saw lying in the wreckage. D Yes, because the bank had provided the pedestal for the artwork.
The husband will recover for his injuries because his wife was among those injured by the collapse of the artwork. The duty to avoid negligent infliction of emotional distress may be breached when the defendant creates a foreseeable risk of physical injury to the plaintiff. In most jurisdictions, a bystander who sees the defendant negligently injuring another can recover for his own distress if (i) the plaintiff and the person injured by the defendant's negligence are closely related, (ii) the plaintiff was present at the scene of the injury, and (iii) the plaintiff personally observed or perceived the event. Observation is typically by sight, but may also be by hearing or other senses under certain circumstances. Here, the husband heard the screams and the sound of breaking glass when the artwork collapsed as he entered the lobby. Even though he evidently did not see the artwork collapse on the diners, he heard it crash where his wife was sitting and saw the immediate aftermath. Because his wife was one of the persons injured by the collapse of the artwork, he can recover damages for the injuries caused by his distress. (A) is incorrect because, as stated above, the majority rule allows a bystander to recover based on the factors stated above even if he is outside the zone of danger of physical injury. (B) is incorrect because, as discussed above, a plaintiff who is present at the scene of the injury may perceive the event by hearing or other senses; under the circumstances here, it was not essential that he observe the actual collapse with his eyes. (D) is incorrect because it does not matter that the bank had provided the pedestal. Even if the negligent construction of the pedestal had been done by a third party, the bank remains liable to invitees on its premises because a business has a nondelegable duty to keep its premises safe for customers.
A dairy farm operated a small processing plant that supplied premium ice cream to nearby specialty shops and ice cream parlors. It entered into a written agreement with a local ice cream parlor to sell "all output" of its Extra Rich ice cream to the ice cream parlor, and the ice cream parlor agreed to sell exclusively the dairy farm's Extra Rich frozen desserts. The agreement stated that the ice cream parlor would pay $25 for each five-gallon container of Extra Rich ice cream that it ordered from the dairy farm. Several months after the parties entered into this contract, demand for high-fat ice creams dropped sharply among the health-conscious consumers who had formerly patronized the ice cream parlor, and the proprietor had to throw out some of its product because the reduced demand meant that opened containers were not used up before the taste of the ice cream became affected. The ice cream parlor wanted to stop selling the dairy farm's Extra Rich ice cream and instead sell a frozen yogurt product produced by another dairy. Can the dairy farm enforce its agreement against the ice cream parlor? response - incorrect A Yes, because changing demand is one of the standard risks of business that both parties assumed. B Yes, because the court will imply a promise on the part of the ice cream parlor to use its best efforts to sell the dairy farm's Extra Rich ice cream. C No, because there was no consideration on the part of the ice cream parlor to support an enforceable contract. D No, because the total price and total quantity terms were never established.
The ice cream parlor has no grounds for avoiding its obligations under the contract with the dairy farm. In effect, the ice cream parlor is advancing the position that its duty to perform under the contract is discharged by impracticability. In contracts for the sale of goods under the UCC, a party's duty to perform may be discharged where performance would be impracticable. Impracticability exists where a party encounters extreme and unreasonable difficulty and/or expense, and such difficulty was not anticipated. Duties will not be discharged where performance is merely more difficult or expensive than anticipated. The facts giving rise to impracticability must be such that their nonoccurrence was a basic assumption on which the contract was made. Where, as here, parties enter into a contract for the sale of goods to be supplied to the public through a retail outlet, both parties must anticipate the possibility that there will be a change in market conditions, resulting in either an increased or decreased demand for the product. Although the decreased demand results in increased expense to the ice cream parlor in performing its contract because of waste, such difficulties arising from changing demand are to be anticipated. Thus, the ice cream parlor does not have the right to no longer buy any of the dairy farm's Extra Rich ice cream. Note that under the UCC, a shutdown by a requirements buyer for lack of orders may be permissible if the buyer is acting in good faith [UCC �2-306, comment 2], but this right would arise only if there were no longer a market for frozen desserts entirely, and that is not the case here. Here, the ice cream parlor simply wants to curtail its losses by selling a more popular type of frozen dessert, which is forbidden by the exclusivity provision. Thus, the ice cream parlor continues to be bound by its duties under the agreement with the dairy farm. (B) is incorrect because, although a court will imply a promise on the part of the ice cream parlor to use its best efforts to sell the dairy farm's products, the facts do not indicate that the ice cream parlor did not use its best efforts. At issue here is whether, despite those efforts, circumstances exist that were unanticipated and now create extreme and unreasonable difficulty or expense for the ice cream parlor in the performance of its contractual duties. (C) is incorrect because the ice cream parlor relinquished its legal right to sell any frozen desserts other than those of the dairy farm. This giving up of a legal right constitutes legal detriment to the ice cream parlor, so there is consideration. (D) is incorrect because an agreement to buy or sell all of one's requirements or output is capable of being made certain by reference to objective, extrinsic facts (i.e., the buyer's actual requirements or the seller's actual output). There is an assumption that the parties will act in good faith; thus, there may not be a tender or demand for a quantity unreasonably disproportionate to a stated estimate or prior output or requirements. Here, the agreement by the dairy farm to sell all of its output of Extra Rich ice cream to the ice cream parlor can be made certain by referring to such factors as the normal output of such product by the dairy farm. In addition, the ice cream parlor agreed to pay $25 for each container of Extra Rich ice cream, so the total price is also ascertainable. Thus, it is not a ground for avoiding enforcement that the total price and quantity were not established.
A fee simple owner of a restaurant provided in his will that the property should go on his death "in fee simple to my friend, but if during my friend's lifetime my son has children and those children are alive when my friend dies, then to said living children." When the owner died, the friend took over the restaurant. If the son has children and one or more of them are alive when the friend dies, who will take title to the restaurant at that time? response - incorrect A The friend's heirs, because the attempted gift to the son's children is invalid under the Rule Against Perpetuities. B The son's children, because their interest is not contingent, being a possibility of reverter. C The son's children, because their interest is vested, subject to defeasance. D The son's children, because their interest will vest, if at all, within a life in being plus 21 years.
The interest given to the son's children does not violate the Rule Against Perpetuities because the interest will vest, if at all, within 21 years after the life of the friend. Pursuant to the Rule Against Perpetuities, no interest in property is valid unless it must vest, if at all, not later than 21 years after one or more lives in being at the creation of the interest. In the case of a will, the perpetuities period begins to run on the date of the testator's death, and measuring lives used to show the validity of an interest must be in existence at that time. Here, the interest given to any of the son's children who are born during the friend's lifetime and who survive the friend must vest, if at all, on the death of the friend (who is a life in being at the time of the owner's death). Thus, this interest will vest, if it does vest, within 21 years after the friend's life, and is therefore not in violation of the Rule Against Perpetuities. (A) is therefore incorrect; if one or more of the son's children is alive at the time of the friend's death, the friend's heirs will get nothing because their fee simple will be divested. (B) incorrectly characterizes the interest of the son's children as a possibility of reverter. A possibility of reverter is the future interest left in a grantor who conveys a fee simple determinable estate. Although under different circumstances the son's children could acquire a possibility of reverter as heirs of the grantor (the owner), their interest in this case was conveyed directly to them in the owner's will. (C) is incorrect because the interest of the son's children is not vested. Their interest is a shifting executory interest rather than a remainder because it divests the fee simple estate of the friend and his heirs. The friend has a fee simple subject to an executory interest because the estate will remain with his heirs if none of the son's children are alive when the friend dies. The friend's death while the son's children are alive divests the interest of the friend's heirs; it is therefore a shifting executory interest rather than a remainder.
A beneficiary has filed a petition in the probate court to contest the validity of a testator's will. The beneficiary contends that when the testator executed the will eight years before, he had a severe mental illness and was incapable of forming a valid testamentary intent. In support of this contention, the beneficiary seeks to offer an affidavit prepared by the testator's former attorney, which states that she was asked to prepare a will for the testator just four months before this will was made. The attorney had refused to do so because it was her opinion that the testator seemed incoherent and paranoid. How should the judge rule on the admissibility of this affidavit? response - incorrect A Admissible. B Inadmissible, as being violative of the attorney-client privilege. C Inadmissible, because it is hearsay not within any exception. D Inadmissible, because it is improper opinion evidence.
The judge should rule this affidavit to be inadmissible hearsay. This affidavit is clearly hearsay, and there is nothing in the facts that shows that it is admissible under any of the exceptions to this rule. Hence, (A) is wrong. (B) is wrong because the observations of the attorney would not be deemed a "communication received from the client." Also, while the attorney-client privilege generally survives the client's death, it does not apply to communications relevant to an issue between parties who are claiming through the same deceased client, such as in the probate proceedings here. (D) is incorrect because a lay person could probably testify to her opinion in this situation since it is rationally based on her own perception, it is helpful to a determination of a fact in issue, and it is not based on scientific, technical, or other specialized knowledge.
A landowner entered into a written agreement with a real estate broker whereby the broker would receive a commission of 10% of the sale price if he procured a "ready, willing, and able buyer" for the landowner's property and if the sale actually proceeded through closing. The broker found a buyer who agreed in writing to buy the property from the landowner for $100,000, the landowner's asking price. The buyer put up $6,000 as a down payment. The agreement between the landowner and the buyer contained a liquidated damages clause providing that, if the buyer defaulted by failing to tender the balance due of $94,000 at the closing date, damages would be 10% of the purchase price. The landowner included that clause because she was counting on using the proceeds of the sale for a business venture that would likely net her at least $10,000. The buyer became seriously ill and defaulted. When he recovered, he demanded that the landowner return his $6,000, and the landowner refused. The broker also demanded the $6,000 from the landowner and was refused. The broker and the buyer filed separate suits against the landowner, with the buyer pleading impossibility of performance. The two cases are consolidated into a single case. How should the court rule as to the disposition of the $6,000? response - incorrect A The landowner keeps the entire $6,000, because the liquidated damages clause is reasonable. B The buyer gets the entire $6,000, because his performance was impossible. C The broker gets the entire $6,000, which is 60% of the commission he is entitled to, because he substantially performed his part of the contract by producing a buyer willing to pay the $100,000 asking price. D The broker gets $600 and the landowner gets $5,400, because the damages clause was reasonable and the broker is entitled to 10% of whatever the landowner realizes from the deal.
The landowner may keep the $6,000 as liquidated damages. A liquidated damages clause is enforceable if: (i) damages are difficult to ascertain at the time of the making of the contract, and (ii) the damages are a reasonable forecast of compensatory damages. Here, the landowner was unsure what her damages would be if she did not receive the sales proceeds from the property, but $10,000 seemed a reasonable amount. Thus, both criteria for valid liquidated damages clauses are met. (B) is incorrect because impossibility must be objective; i.e., performance cannot be accomplished by anyone. Physical incapacity of a person necessary to effectuate the contract may discharge contractual duties if that person's performance is clearly impossible. (Usually this occurs in personal services contracts, where only that one person can perform the required duty.) Although the buyer was seriously ill, it is not clear that this made it impossible for him to produce the $94,000. Without more facts, it is reasonable to assume that someone else could have delivered the money or that his mortgage would still have gone through, etc. (C) is incorrect because the conditions for the broker's payment were not met: It is debatable whether he produced a "ready, willing, and able" buyer, and in any event the sale did not actually proceed through closing. (D) is incorrect because the broker was to receive proceeds from the sale of the property; the $6,000 was damages and not sale proceeds.
A landowner possessed a 40-acre tract of land. He had inherited 30 acres and had possessed the other 10 acres for longer than the statutory period necessary to acquire title by adverse possession from a rancher. The landowner entered into a land sale contract promising to convey the 40 acres to a developer. The contract provided that the landowner would convey marketable title. The developer paid the landowner the purchase price and accepted a deed from him. The developer promptly recorded the deed. The rancher, having learned of the sale, brought a successful action against the developer to quiet title. The developer realized for the first time that there were no covenants for title in his deed. The developer brings an action against the landowner. What is the most likely outcome of the suit?
The landowner will win because the terms of the deed, not of the contract, control his liability. There is an implied covenant in every land sale contract that at closing the seller will provide the buyer with a title that is "marketable." Marketable title is title reasonably free from doubt, i.e., title that a reasonably prudent buyer would be willing to accept. It need not be a "perfect" title, but the title must be free from questions that might present an unreasonable risk of litigation. Generally, this means an unencumbered fee simple with good record title. Generally, a title acquired by adverse possession is not considered marketable because the purchaser might be later forced to defend in court the facts that gave rise to the adverse possession against the record owner. Here, the marketability requirement did not have to be implied, it was an express term of the contract. Under the doctrine of merger, the contract merges into the deed, and the terms of the contract are meaningless. Even though the contract specified a "good and marketable title," it is the deed that controls, and the deed contained no covenants of title. A deed does not incorporate the title terms of a contract. Thus, (A) is wrong. (B) is wrong; it is not supported by the facts. (D) is wrong because the developer's negligence is irrelevant.
A landlord owned a prestigious downtown office building. A law firm leased the entire building from the landlord for a term of 20 years. The lease included a provision that taxes on the building would be paid by "the lessee, his successors, and assigns." The law firm occupied the building and paid the rent and taxes for eight years. At the end of the eight-year period, the law firm assigned the balance of the lease to an accounting firm and vacated the premises. The assignment was written, but there was no provision concerning the accounting firm's assumption of the duties under the lease. The accounting firm occupied the building and paid the rent and taxes for five years. At the end of the five-year period, the accounting firm subleased the building for five years to an investment company and vacated the premises. The sublease was written, but there was no provision concerning the investment company's assumption of the duties under the lease. The investment company now occupies the building and has paid the rent but not the taxes. The landlord has sued all three (i.e., the law firm, the accounting firm, and the investment company) for failure to pay the taxes. The landlord should prevail against whom?
The law firm and the accounting firm are liable. After an assignment, the original tenant is no longer in privity of estate with the landlord. However, a tenant may still be held liable on its original contractual obligations to the landlord on privity of contract grounds. Here, the law firm is liable because it made the original deal with the landlord, which included the obligation to pay taxes on the building. The law firm remains in privity of contract with the landlord throughout the term of the lease unless it is otherwise discharged. In an assignment, the assignee stands in the shoes of the original tenant in a direct relationship with the landlord. Each is liable to the other on all covenants in the lease that run with the land, which would include the obligation of the lessee to pay taxes on the property. Here, the accounting firm is liable because as an assignee it is in privity of estate with the landlord. The accounting firm remains in privity of estate until it assigns to someone else. The sublease to the investment company is not an assignment. A sublessee is not personally liable to the landlord for rent or for the performance of any other covenants made by the original lessee in the main lease (unless the covenants are expressly assumed) because the sublessee does not hold the tenant's full estate in the land (so no privity of estate). Here, the investment company is not liable because, as a nonassuming sublessee, it is not in privity of contract or estate with the landlord. Therefore, (B) is the correct choice, and (A), (C), and (D) are wrong.
A state statute defines all murders as second degree murders unless deliberation and premeditation can be shown, in which case the crime is elevated to first degree murder. Manslaughter is defined as at common law. The defendant, just having been served with divorce papers, decided to drown his sorrows at the local pub. After drinking heavily and becoming very intoxicated, the defendant became enraged when another patron spilled a drink on him. He took a nearby ashtray and smashed it over the patron's head, killing him instantly. The crimes below are listed in descending order of seriousness. What is the most serious crime of which the defendant could be convicted? response - incorrect A Murder in the first degree. B Murder in the second degree. C Voluntary manslaughter. D Involuntary manslaughter.
The most serious crime that the defendant would be convicted of is murder in the second degree. The jurisdiction defines murder in the first degree as deliberate premeditated murder, whereas all other types of killings are defined as at common law. Deliberation and premeditation requires some time of cool reflection on the idea of killing. In the instant case, there are no facts indicating that the defendant coolly reflected on the idea of killing. The facts indicate an impulsive killing rather than any type of deliberate, premeditated killing. Furthermore, the facts indicate that the defendant was very intoxicated, which would serve as a basis for reducing the crime to second degree murder. Thus, (A) is incorrect. The state defines murder in the second degree just like common law murder. At common law, murder required malice; i.e., (i) the intent to kill, (ii) the intent to inflict great bodily injury, (iii) reckless indifference to an unjustifiably high risk to human life, or (iv) the intent to commit a felony. Here, the reckless indifference element arguably could be satisfied. By smashing a heavy ashtray over the other patron's head, the defendant unjustifiably disregarded that the blow could be a killing blow. Neither would intoxication be a defense, as there would be no specific intent-to-kill requirement under this type of analysis. (D) is incorrect. Involuntary manslaughter is a killing committed with criminal negligence or during the perpetration of some unlawful act not encompassing a felony for felony murder. Certainly, striking another with a heavy object would constitute criminal negligence sufficient for conviction. However, involuntary manslaughter is a lesser crime than murder in the second degree, and the call of the question asks for the most serious crime of which the defendant could be convicted, making (B) a better choice than (D). (C) is also incorrect. Voluntary manslaughter is a killing committed under the duress of an adequate provocation, and it requires (i) a provocation sufficient to arouse the sudden and intense passion in the mind of an ordinary person such as to cause him to lose self-control; (ii) the defendant to be in fact provoked; (iii) an insufficient time to cool off; and (iv) the defendant did not in fact cool off. It is unlikely that having a drink spilled on him would cause an ordinary person to commit murder. As a result, such a provocation is not adequate to reduce the killing to voluntary manslaughter.
A boy mowing his lawn noticed a strong vibration from the engine but continued to mow. The engine housing suddenly broke apart and pieces flew off the lawnmower. One piece struck the boy in the head, seriously injuring him. The boy's mother was inside the house and heard yelling from the backyard. She went to the window and saw her son lying on the ground by the lawnmower and a friend of his kneeling over him. She became very upset and fainted. Subsequent investigation showed that a negligent repair by a local mechanic caused the engine housing to shatter. The mother brought a lawsuit against the mechanic, seeking recovery for her son's injury and the emotional distress she suffered. Can the mother recover damages for her emotional distress?
The mother cannot recover damages for her emotional distress. A duty to avoid negligent infliction of emotional distress may be breached when the defendant creates a foreseeable risk of physical injury to the plaintiff. The mother might try to assert two theories in support of her emotional distress claim, but she is unlikely to prevail on either one. First, she could claim distress flowing from fear for her own safety, but she may prevail only if the defendant's negligence placed her in a zone of danger. Here, because she was safely inside her home and quite distant from the mower when it exploded, she was not in a zone of danger. Alternatively, she could claim distress flowing from her anguish at seeing her son injured. However, for a bystander who is outside the zone of danger from the risk of physical injury but who suffers emotional distress from seeing the defendant negligently injure another, most states allow recovery only if: (i) the plaintiff and the person injured by the defendant are closely related; (ii) the plaintiff was present at the scene of the injury; and (iii) the plaintiff personally observed or perceived the event. Here, while she is related to her son, who was injured by the mechanic's negligence, she was not present at the scene of the injury and did not personally observe or perceive the event. Hence, she cannot recover damages for negligent infliction of emotional distress. (A) is incorrect. Under proximate cause rules, a third party's negligence that contributes to the plaintiff's harm will not be considered a superseding force where it is within the foreseeable risk created by the defendant's negligence. While the son may have been negligent in continuing to use the mower, this conduct would not cut off the defendant's liability to the mother if she otherwise could recover for her distress. (C) is incorrect because the plaintiff's close relationship with the injured person is only one of the requirements for a plaintiff outside the zone of danger to recover emotional distress damages. Because the mother was not present at the scene and did not observe the event, she cannot recover. (D) is incorrect. While the mother can recover on her son's behalf for his injury, she cannot recover for her emotional distress, as discussed above.
The owner of an old car parked it in front of his house with a "for sale" sign in the windshield. In response to an inquiry from his neighbor, the car owner said that he would take $400 for the car. The neighbor responded, "You've got a deal." Because it was a Sunday, and the banks were closed, the neighbor told the car owner that he would come to his house with the $400 the next day at about 6 p.m. The car owner said that was fine. At 9:15 the next morning, the car owner called his neighbor and told him that when they had talked the previous day, he forgot that he had just put two new tires on that car and that he would need an extra $50 to cover their cost. The neighbor agreed to bring $450 in cash to the car owner's house at about six o'clock. Is the neighbor legally bound to pay the car owner the additional $50? response - incorrect A Yes, because the original contract was not in writing. B Yes, because the contract, as modified, does not need to be in writing. C No, because no additional consideration was given for the oral modification. D No, because neither the neighbor nor the car owner is a merchant.
The neighbor must pay the car owner the additional $50 because the parties have an enforceable contract. A contract for the sale of goods (the car) was formed when the neighbor said, "You've got a deal." The parties then orally agreed to a modification of the contract when the car owner called his neighbor the next morning. Under the Statute of Frauds provision in the UCC, which applies to all contracts for the sale of goods, a promise requires a writing signed by the party to be charged to be enforceable if it is for the sale of goods of $500 or more. Here, the contract as modified is under $500, so it is enforceable even though it is not in writing. (A) is incorrect because the fact that the original contract was not in writing is irrelevant to the issue of whether the modified contract is enforceable. If the modification had caused the contract to reach or exceed $500, the car owner could not have collected the additional $50 from his neighbor. (C) is incorrect because under UCC section 2-209, no consideration is needed for a good faith modification of a contract for the sale of goods. (D) is incorrect because the UCC rules on modifications and the Statute of Frauds apply to all contracts for the sale of goods, not just those between merchants.
A victim was struck by a car in a hit-and-run accident. A police officer arrived half an hour after the accident. The victim was in shock and came in and out of consciousness. As the officer applied first aid, the victim muttered, "I know I'm going to die. Oh my, he ran the light!" The victim fell back into unconsciousness, but revived again and muttered, "Why didn't he stop?" The officer heard the comments clearly and made a note of them. Good police work by the officer and others led to the discovery of the driver of the car that struck the victim. The victim survived and filed a tort action against the driver. Before the case came to trial, the victim died of a heart attack. The causes of the heart attack were totally unrelated to the accident. The laws of the jurisdiction allow for survival of personal injury actions. Thus, the victim's estate is substituted for the victim as plaintiff. If the plaintiff's attorney seeks to have the officer testify to the victim's statements at the time of the accident, how will the court rule? response - incorrect A Inadmissible, because the victim did not die as a result of the accident. B Inadmissible, because this is a civil case and not a criminal matter. C Admissible, because the victim's statements were present sense impressions. D Admissible, because the statements were made at a time when the victim feared impending death.
The officer's testimony as to the victim's statements is admissible because the statements were made when the victim feared impending death and so they qualify under the dying declaration exception to the hearsay rule. Hearsay is a statement, other than one made by the declarant while testifying at the trial or hearing, offered in evidence to prove the truth of the matter asserted. [Fed. R. Evid. 801(c)] Upon appropriate objection, a hearsay statement to which no exception is applicable must be excluded. Under the dying declaration exception to the hearsay rule, a statement made by a now-unavailable declarant while believing her death was imminent that concerns the cause or circumstances of what she believed to be her impending death is admissible. [Fed. R. Evid. 804(b)(2)] The declarant need not actually die as a result of the circumstances giving rise to her belief of imminent death. Here, testimony as to the victim's statements would be hearsay, because they are out-of-court declarations offered for the truth of the matter asserted; i.e., that the driver of the car that hit her ran a red light. However, these statements related to the circumstances of what the victim believed to be her impending death and the victim (who is now unavailable due to her death) made these statements under a fear of imminent death, as indicated by her condition and her statement "I know I'm going to die." Consequently, all of the elements of the dying declaration exception are present, and the officer's testimony as to the statements is admissible. (A) is incorrect because the declarant need not actually die as a result of the incident that gives rise to the statements. Indeed, the declarant need not die at all. All that is required is that the declarant be unavailable at the time the statements are offered. (B) is incorrect because it reflects the traditional view, which limited the admissibility of dying declarations to homicide cases, rather than the position of the Federal Rules, which allow such declarations in both civil cases and homicide prosecutions. (C) is incorrect because the statements do not qualify under the present sense impression exception to the hearsay rule. A present sense impression is a statement that describes or explains an event or condition, and is made while or immediately after the declarant perceives the event or condition. Here, the victim's statements were made at least one half-hour after the accident. This time lapse between the accident and the statements means that such statements were not made either at the time the victim received a sense impression or immediately thereafter; thus, the present sense impression exception is inapplicable to these facts.
During a presidential campaign, a candidate's campaign manager secretly engaged in activities that may have violated both state and federal laws. After the candidate was elected President, the FBI investigated the manager's activities as well as whether the President was involved. After the campaign manager was indicted in federal court, but before trial, the President granted a blanket pardon to the campaign manager for "all federal crimes that may have been committed in the past 20 years." Is the pardon valid? response - incorrect A Yes, because the pardon power is an unqualified power (except as to impeachment). B No, because a presidential pardon that interferes with an inquiry into the President's own actions constitutes an obstruction of justice. C No, presidential pardons must relate to specific crimes; the President cannot issue blanket pardons. D No, because a President's pardon power is limited to crimes that allegedly took place while the President is in office.
The pardon is valid. Article II, Section 2 of the United States Constitution grants the President the power to grant reprieves and pardons for offenses against the United States, except in cases of impeachment. This pardon power is not subject to control by Congress, and it includes the power to commute a sentence on any conditions the President chooses (as long as the conditions do not offend some other constitutional provision). (B) is incorrect because even if the action of issuing the pardon amounted to the crime of obstruction of justice (a questionable assumption), the pardon itself would not be invalidated. The power to pardon is a constitutional power, superior to laws found in statutes. (C) is incorrect because the pardon power is not so limited. Blanket pardons are valid. (D) is also incorrect because the pardon power is not so limited. Presidents may pardon offenses that occurred before the President took office.
A pedestrian was injured in an auto accident caused by a driver. The pedestrian's injuries included a broken nose and a broken toe. Not sure of the strength of her case, she sued the driver only for the injuries to her nose. She was awarded $15,000 in damages. Encouraged by this success, she now wishes to sue the driver for the injuries to her toe. May she sue the driver again? response - incorrect A No, because all related claims "merged" with the final decision in the first case. B No, because she will be collaterally estopped from pursuing the claim. C Yes, and she may use the prior decision against the driver. D Yes, but she will have to relitigate the driver's liability.
The pedestrian may not sue the driver again because of merger. Merger occurs when the plaintiff wins; her cause of action is said to "merge" into the judgment such that she cannot relitigate the cause of action later. The court would hold that all of the personal injuries received by the pedestrian in one accident constitute a single cause of action, and that claim preclusion principles forbid relitigation. (B) is incorrect. Issue preclusion, also called "collateral estoppel," applies to issues, not entire cases. (C) and (D) are incorrect for the reasons stated above.
A breeder of quarter horses entered into an agreement with a rancher to sell and deliver two quarter horses, one to the rancher and the other to the rancher's fianc�e as a gift. Although the fair market value of each horse was $3,000, the horse breeder agreed to sell both horses together for a total price of $5,000. Under the agreement that the rancher wrote out and both parties signed, the horse breeder agreed to deliver one horse to the rancher on August 1, at which time the rancher agreed to pay the horse breeder $5,000. The horse breeder further agreed to deliver the other horse to the rancher's fianc�e on August 12. On August 1, the horse breeder delivered the first horse to the rancher and, at the same time, the rancher gave the horse breeder a certified check for $5,000. On August 12, the horse breeder brought the second horse to the residence of the rancher's fianc�e and told her that the horse was a gift from the rancher. The rancher's fianc�e told the horse breeder that she loathed quarter horses and she refused to take the horse. The horse breeder brought this horse back to his farm and sent an e-mail to the rancher, informing him that his fianc�e refused delivery and that he (the horse breeder) could not keep the horse. Two weeks later, after not hearing from the rancher, the horse breeder sold the horse to an interested party for $3,000. If the rancher sues the horse breeder, how much should the rancher recover? response - incorrect A $3,000, the value of the second horse. B $2,000, the difference between the value of the horse delivered to the rancher and what the horse breeder received from the rancher. C Nothing, because the rancher was not financially harmed. D Nothing, because the horse breeder performed his part of the contract.
The rancher should recover $2,000 because that is the amount by which the horse breeder would be unjustly enriched. In a proper tender of delivery under UCC section 2-503, the seller must put and hold conforming goods at the buyer's disposition for a time sufficient for the buyer to take possession. The seller must give the buyer notice reasonably necessary to enable him to take possession of the goods. Proper tender of delivery entitles the seller to acceptance of the goods and to payment according to the contract. [UCC �2-507] Having made a proper tender of delivery at the place designated by the rancher and having notified the rancher of his fianc�e's nonacceptance, the horse breeder has discharged his duty under the contract. When a party's duty of performance is discharged, the other party is entitled to restitution of any benefits that he has transferred to the discharged party in an attempt to perform on his side. With the horse breeder's contractual duty to deliver the second horse to the rancher's fianc�e discharged, the horse breeder would be unjustly enriched, to the detriment of the rancher, if he were permitted to keep the entire $5,000 paid to him by the rancher. The rancher conferred a benefit upon him by paying him $5,000 in exchange for two horses, one of which was to be delivered to the rancher, the other to the rancher's fianc�e. Because delivery to the fianc�e cannot be accomplished, the rancher finds himself in a position of having paid $5,000 for one horse, the fair market value of which is $3,000. Thus, if the horse breeder is permitted to retain the sum of $5,000, he will be unjustly enriched by $2,000. Therefore, the rancher should recover restitution of $2,000. (A) is incorrect because $3,000 represents more than the amount by which the horse breeder has been unjustly enriched. Although the value of the second horse is $3,000, keep in mind that the horse breeder's duty to deliver the horse to the rancher's fianc�e has been discharged (and the horse breeder still has title to the horse under the UCC rule that title passes on delivery). The rancher received a discount of $1,000 off the total fair market value of the two horses because he was buying both of them. Once the horse breeder's duty under the contract is discharged, the rancher cannot recover the benefit of that bargain under the contract; he can only recover the benefit conferred upon the horse breeder, the retention of which would unjustly enrich the horse breeder. Because the horse breeder has received $5,000 from the rancher for one horse worth $3,000, the amount of unjust enrichment is $2,000. (C) is incorrect because, if the rancher recovers nothing, he will have incurred financial harm by paying $5,000 for one horse worth $3,000. (D) is incorrect because the fact that the horse breeder tendered performance but was unable to complete delivery of the second horse to the rancher's fianc�e, solely due to her refusal to accept the horse, does not justify the horse breeder's keeping the entire $5,000 paid by the rancher, because the horse breeder would be unjustly enriched.
An automotive engineer announced that he had developed a carburetor that will enable cars to achieve 100 miles per gallon of fuel, and that he will allow the carburetor to be inspected next month. Soon after, a former employer of the engineer brought an action to prohibit the engineer from displaying the carburetor, claiming that the engineer probably had stolen the carburetor's design from the employer. The court granted the employer a temporary restraining order prohibiting the engineer from disclosing any mechanical details of his carburetor, and ordered a hearing to be held in one week to determine whether a preliminary injunction should be issued. Because each party would have to reveal the mechanical details of his designs at the hearing, the employer requested that the hearing be closed to the public and that the record be sealed to avoid revelation of his designs. The court granted the request. A reporter for a monthly automobile magazine heard about the case and wanted to attend the hearing. When he was told that the hearing would be closed, he filed an action to have it opened. What is the reporter's best argument for opening the hearing? response - incorrect A Closure is not necessary to preserve an overriding interest here. B The right of freedom of the press is extensive and allows the press to attend all hearings of interest to the public. C Closure here amounts to a prior restraint. D Under the fairness doctrine, the magazine will be required to give each litigant an opportunity to present his side of the case.
The reporter's best argument is that the closure here is not necessary to preserve an overriding interest because trials and pretrial hearings generally must be open to the public. The Supreme Court has held, at least in the context of criminal cases, that trials and pretrial proceedings can be closed only if closure is necessary to preserve an overriding interest and the closure order is narrowly tailored to serve the overriding interest. While the Court has not yet established the standard for civil matters such as the case here, several Justices and commentators have suggested that the same standard will be applied in civil cases since they too have historically been open to the public. (B) is a false statement of the law—freedom of the press is not absolute and does not allow the press unlimited access to any hearing of interest to the public; the hearing may be closed where an overriding interest in protecting the privacy of the parties is established. (C) is not as good an argument as (A) because, while closure here would amount to a prior restraint (a court order or administrative system that keeps speech from occurring), the prior restraint would be justified if the government proves that it was narrowly tailored to achieve a compelling interest. The argument in (A) negates this possibility and so is a better argument. (D) is incorrect because the fairness doctrine is irrelevant to the issue of whether a hearing should be open to the public. It was a rule of the Federal Communications Commission that required, among other things, that the media give political candidates an opportunity to oppose candidates or views endorsed by the media.
A city's water board election laws provide that, although members of the board are elected at large, one member of the board is required to live within each of the five designated water districts within the city. The city's population was more or less evenly distributed among the districts when this election law was enacted. A resident and registered voter of the city investigated the district residency requirement and discovered that most of the city's newer residents had moved into the same two water districts, so that the city's population was no longer evenly distributed among the five water districts. Instead, 80% of the city's residents lived within its central and eastern water districts, while the other 20% of the city's residents were scattered among its three other, more rural, districts. If the resident files suit in federal court challenging the constitutionality of the residency requirement, how will the court most likely rule? response - incorrect A The residency requirement is unconstitutional because it impairs the voters' equal protection rights, in that it gives the voters in the less populous districts more effective representation on the water board. B The residency requirement is unconstitutional because it violates the candidates' equal protection rights. C The residency requirement is constitutionally permissible because the water board members do not exercise legislative power. D The residency requirement is constitutionally permissible because the water board members are elected at large.
The residency requirement is permissible because the water board is elected at large. The Equal Protection Clause prohibits state dilution of the right to vote, so that when a governmental body establishes voting districts for the election of representatives, the number of persons in each district may not vary significantly. This is known as the principle of "one person, one vote." This principle applies to almost every election where a person is being elected to perform normal governmental functions (e.g., an election for trustees for a junior college district). However, the principle of one person, one vote generally is inapplicable where there is an at-large system of election (except where the system is adopted for discriminatory purposes). Here, the water board members are elected by all of the qualified voters in the city in an at-large system (rather than having the voters of each individual district select one board member apiece), and no discriminatory intent is evident. Thus, the statutory provision requiring board members to reside in each of the five districts does not result in an imbalance or a dilution of the voting rights of the citizens of the city. Consequently, (A) is incorrect, and (D) presents an accurate statement of the constitutionality of the residency requirement. (Note that the answer might be different under federal statute because the city would have to prove a valid, nondiscriminatory purpose.) (C) is incorrect even though it reaches the correct result. While the Supreme Court has exempted special purpose water storage districts from the one person, one vote requirement, the basis of the decision was the specialized nature of the entity. The constitutional requirements apply not only to legislators, but also to elected administrative and executive officials. (B) is incorrect because, even assuming that the residency requirement violates the candidates' equal protection rights, the resident would not have standing to raise the issue. Generally, a claimant must assert his own constitutional rights and cannot assert the rights of third parties.
A retailer entered into a written contract with a wholesaler whereby the wholesaler agreed to sell, and the retailer agreed to buy, 100 boxes of sunglasses manufactured by a large corporation located in a neighboring city. The agreed-upon price was $75 per box. Two weeks before the specified delivery date, the wholesaler told the retailer that it would not be able to fill its order, because of unexpected high demand for sunglasses this season. Although the retailer learned that the needed quantity of the same brand of sunglasses could be shipped within two days for $83 per box from a supplier in another area, the retailer instead purchased 100 boxes of the sunglasses locally at a cost of $90 per box. These sunglasses were of a slightly higher quality than the sunglasses that were originally contracted for. A few days before the original delivery date, the wholesaler notified the retailer that it would fill the order, and tendered 100 boxes of the sunglasses on the date of delivery. However, the retailer refused to accept them. At that time, the wholesale market price of the sunglasses had declined to $80 per box. If the retailer sues the wholesaler for damages based on the wholesaler's alleged breach, what is the retailer likely to recover? response - incorrect A $1,500, the difference between the cost of cover and the contract price. B $800, the difference between the contract price and the nonlocal supplier's price. C $500, representing the difference between the contract price and the wholesale market price at the time of performance. D Nothing, because the retailer obtained cover without waiting a commercially reasonable time for the wholesaler to retract the repudiation.
The retailer is entitled to recover $800. The wholesaler's notice that it would be unable to fill the retailer's order constituted an anticipatory repudiation, which the retailer was entitled to treat as a total breach. Under the UCC, the buyer's basic remedy where the seller breaches by refusing to deliver is the difference between the contract price and either the market price or the cost of buying replacement goods ("cover"). If the buyer intends to fix damages based on the latter measure, the buyer must make a reasonable contract for substitute goods in good faith and without unreasonable delay. Here, the retailer chose to make a contract for a higher quality of sunglasses at a higher price, even though the model that he had originally ordered was available from a supplier outside the area. While the retailer need not find the lowest available price in the country or make a contract for substitute goods with an unreliable supplier, he was aware that he could have obtained the sunglasses in plenty of time from the nonlocal supplier. Absent additional facts that would justify the retailer's decision, he can recover only the difference between the contract cost and a reasonable contract for substitute goods. Hence, (A) is wrong because the retailer's contract for cover probably would not be deemed to be commercially reasonable. (C) is wrong because the retailer's remedy based on market price would be determined at the time the retailer learned of the breach, not necessarily the time of performance. In the case of an anticipatory repudiation such as this, the buyer may either treat the anticipatory repudiation as a total breach and pursue his breach of contract remedies, or suspend his performance and await the seller's performance for a commercially reasonable time. The retailer chose to treat the wholesaler's notice as a total repudiation and breach of contract. Hence, the market price remedy would be measured at that time because that is when the retailer "learned of the breach," rather than at the time of performance. (D) is wrong because the nonrepudiating party need not wait for the repudiating party to retract its repudiation. The retailer exercised its option to treat the repudiation as a total breach and buy substitute goods. Once that occurred, the wholesaler was not entitled to retract its repudiation and force the retailer to accept the sunglasses.
A seller entered into a written contract to sell a tract of land to a buyer. The buyer was to pay $1,500 per month for five years, at which time the seller would deliver a warranty deed. The contract was silent as to the quality of title to be conveyed. After making 12 payments, the buyer discovered that a neighbor had an easement of way over the land, which was not discussed at the time the seller and buyer entered into the contract. The neighbor had not used the easement over the previous year because she had been out of the country. On the basis of the easement, the buyer wishes to cancel the contract. Which party is more likely to prevail? response - incorrect A The seller, because the neighbor's easement has been extinguished. B The seller, because the buyer has no basis on which to rescind the contract. C The buyer, because the obligation to convey marketable title is implied. D The buyer, because the seller has breached the covenant against encumbrances.
The seller is more likely to prevail because the buyer has no basis on which to rescind the contract. Absent a provision to the contrary, a contract for the sale of land contains an implied promise by the seller that she will deliver to the buyer a marketable title at the time of closing. This promise imposes on the seller an obligation to deliver a title that is free from reasonable doubt; i.e., free from questions that might present an unreasonable risk of litigation. Title is marketable if a reasonably prudent buyer would accept it in the exercise of ordinary prudence. An easement that reduces the value of the property (e.g., an easement of way for the benefit of a neighbor) generally renders title unmarketable. If the buyer determines, prior to closing, that the seller's title is unmarketable, he must notify the seller and allow a reasonable time to cure the defect. If the seller is unable to acquire title before closing, so that title remains unmarketable, the buyer can rescind, sue for damages caused by the breach, or obtain specific performance with an abatement of the purchase price. However, the buyer cannot rescind prior to closing on grounds that the seller's title is unmarketable. Where an installment land contract is used, the seller's obligation is to furnish marketable title when delivery is to occur, e.g., when the buyer has made his final payment. Thus, a buyer cannot withhold payments or seek other remedies on grounds that the seller's title is unmarketable prior to the date of promised delivery. Here, there is a valid easement on the property (see below), but the seller has four years in which to cure this defect. Thus, the buyer cannot yet rescind on grounds that title is unmarketable. (A) is incorrect because the neighbor's easement has not been extinguished. An easement can be extinguished where the owner of the privilege demonstrates by physical action an intention to permanently abandon the easement. Mere nonuse is not sufficient to terminate an easement, unless the nonuse is combined with other evidence of intent to abandon it. Here, the fact that the neighbor did not use the easement for a year because she was out of the country does not establish her intent to abandon the easement. (C) is incorrect because, although the law implies in every land sale contract a covenant that title will be marketable, the seller has until the time of delivery to cure the defect. (D) is incorrect because the deed has not yet been delivered. The covenant against encumbrances is a covenant contained in a general warranty deed which assures that there are neither visible encumbrances (e.g., easements) nor invisible encumbrances (e.g., mortgages) against the title or interest conveyed. This covenant is breached, if at all, at the time of conveyance. Here, the deed has not yet been delivered, and thus this covenant has not yet been breached.
A skier broke his leg when he was knocked down by the chair lift as he tried to avoid other skiers who had fallen off while disembarking. The ski resort employee operating the lift had not been paying attention and had failed to stop the lift. Ski patrol personnel placed the skier on a stretcher, which they then hooked up to a snowmobile to bring him down the mountain. The route down ran along the edge of a ski trail. Midway down, a novice snowboarder tried to see how close he could come to the stretcher without hitting it, but he lost control and landed on top of the skier's leg, damaging it further. The skier filed a lawsuit against the snowboarder and the resort in a jurisdiction that has adopted a comparative contribution system in joint and several liability cases. At trial, the skier's physician testified that the skier's leg was permanently disabled, but that neither injury, by itself, would have caused the permanent disability and it was impossible to quantify how much each injury contributed to the disability. The jury determined that the damages from the permanent disability equaled $2 million, and that the snowboarder and the resort were each 50% at fault. What amount of damages can the skier recover from the snowboarder for his permanent disability? response - correct A $1 million, because the jurisdiction follows comparative contribution rules. B $2 million, because it was not possible to identify the portion of the injury that the snowboarder caused. C Nothing for his permanent disability, because the skier has not met his burden of proof as to the amount of damages that the snowboarder caused. D Nothing for his permanent disability, because the injury inflicted by the snowboarder, by itself, would not have caused the disability.
The skier can recover $2 million from the snowboarder because the snowboarder is jointly and severally liable for the injury. The doctrine of joint and several liability provides that when two or more tortious acts combine to proximately cause an indivisible injury to plaintiff, each tortfeasor will be jointly and severally liable for that injury. This means that plaintiff can recover the entire amount of his damages from any one defendant. The doctrine applies even though each tortfeasor acted entirely independently and at different times. Here, both the snowboarder and the employee of the ski resort breached their duty to the skier to act with reasonable care. Each tortfeasor's act was the actual cause of the skier's disability because but for either one of the acts, his leg would not have been permanently disabled. The snowboarder's act was the proximate cause of the skier's disability because the disability was the direct result of the snowboarder's act. The fact that the extent of the harm was unforeseeable is irrelevant; i.e., the tortfeasor takes the victim as he finds him. Thus, the skier can recover the entire $2 million from the snowboarder. (A) is incorrect because the contribution rules govern only whether a defendant required to pay more than his share of damages has a claim against the other jointly liable parties for the excess. Contribution does not involve the amount of damages that the plaintiff can collect in the first place. (C) is incorrect because the skier has met his burden of proof by establishing that the snowboarder was an actual and proximate cause of his permanent disability. Because the injury caused by the tortfeasors was not divisible, under joint and several liability rules, the snowboarder is liable for the full amount of the damages, including that attributable to the permanent disability. (D) is incorrect because but for the snowboarder's collision with the skier, the skier would not have been disabled. The "but for" test applies in concurrent cause situations—cases where several acts combine to cause the injury, but none of them standing alone would have been sufficient. The fact that the snowboarder's act standing alone would not have caused the disability is irrelevant to the snowboarder's liability.
A landowner validly conveyed a small office building to the Green Party "as long as they use it for operating quarters until the next presidential election." After the next presidential election, which was in three years, the building would go to a private organization that monitors and prepares comprehensive listings of gas prices throughout the country. A year after the conveyance, the landowner died, validly devising all of her property to her son. Although this jurisdiction is a common law jurisdiction with respect to all real property considerations, the state's probate laws provide that future interests or estates in real property may be passed by will or descent in the same manner as present or possessory interests. Last week, the Green Party and the gas monitoring organization joined together to sell the office building in fee simple absolute to a developer. The son filed suit to prevent the sale of the property to the developer. In this action, who should prevail? response - incorrect A The Green Party and the gas monitoring organization, because together they own a fee simple absolute in the building. B The Green Party and the gas monitoring organization, because the attempted restrictions on the use of the property violate the Rule Against Perpetuities. C The Green Party and the gas monitoring organization, because the deed restriction was an unlawful restraint on alienation. D The son, because he did not sign the contract of sale.
The son may enjoin the sale because he has an interest in the property. A fee simple determinable is an estate that automatically terminates on the happening of a stated event. The Green Party's interest in the office building is a fee simple determinable because it lasts as long as the Party is using the building for operating quarters. However, the grant does not provide for the contingency of the Green Party ceasing to use the building as operating quarters before the next presidential election. This gap would be filled by a possibility of reverter retained by the landowner. Because the landowner passed that interest to her son in her will, there can be no contract to sell the property without his signature. Note: Although the gas monitoring organization appears to have an indefeasibly vested remainder (i.e., it is created in an ascertained company, is certain to become possessory, and is not subject to being defeated, divested, or diminished in size), its interest is not capable of taking on the natural termination of the preceding estate and so is characterized as a springing executory interest. (A) is wrong because the son also has an interest in the land. (B) is wrong because the interest in the office building will pass to the gas monitoring organization, if at all, within 21 years. (C) is wrong because the Green Party is not prohibited from transferring any interest; it could pass a defeasible fee.
Congress enacted a statute that purported to ban all discrimination against African-Americans in any commercial transaction taking place within the United States. Would the statute most likely be held constitutional? response - incorrect A Yes, under Thirteenth Amendment provisions barring badges or incidents of slavery. B Yes, because the federal government has an important interest in furthering the equal protection provisions of the Fourteenth Amendment. C No, because Congress's powers under the Commerce Clause do not extend so far as the statute would require. D No, because commercial transactions are not among the privileges or immunities of national citizenship.
The statute is constitutional as a legitimate exercise of congressional enforcement powers under the Enabling Clause of the Thirteenth Amendment. The Thirteenth Amendment prohibits slavery. The Enabling Clause of the amendment has been held to confer on Congress the authority to proscribe almost any private racially discriminatory action that can be characterized as a badge or incident of slavery. Because the statute at issue bans all discrimination against African-Americans in commercial transactions, it necessarily reaches private conduct. Such congressional action is constitutionally permissible pursuant to the Thirteenth Amendment. (B) is incorrect. Application of the Fourteenth Amendment has been limited to cases involving state action. [See United States v. Morrison (2000)] The statute here reaches private action, and so the Thirteenth Amendment is the correct source for the law, since that amendment addresses private action. (C) is incorrect because, even if Congress's power over interstate commerce would not reach every commercial transaction, the statute would be enforceable under the Thirteenth Amendment, as discussed above. (D) is incorrect because it is irrelevant. While it is true that the commercial transactions here are not among the privileges or immunities of citizenship (which include rights such as the right to petition Congress for redress and the right to interstate travel), the law can be based on the Commerce Clause or the Thirteenth Amendment, and thus is constitutional.
A defendant is on trial for murder. The only evidence linking the defendant to the crime is some blood found at the scene. The lead detective testifies that an officer took a vial containing a blood sample that had been retrieved by a crime scene technician and drove off with it. The officer is now dead. Next, the prosecution presents as a witness a crime lab chemist. The chemist will testify that he took a vial of blood that contained a label identifying it as having been retrieved from the subject crime scene, and that he performed tests that established a match between that blood and a blood sample taken from the defendant. Is the testimony of the chemist admissible? response - incorrect A Yes, because there has been proper authentication. B Yes, because the chemist qualifies as an expert witness. C No, because there is insufficient evidence of chain of custody. D No, because he did not take the original blood sample at the scene of the crime.
The testimony is inadmissible because it has not been shown what happened to the blood between the time the officer took it and the time the chemist examined it. Real evidence presents an object in issue directly to the trier of fact. One of the general requirements for admissibility of real evidence is that it be authenticated; i.e., that it be identified as being what its proponent claims it is. If the evidence is of a type that is likely to be confused or can be easily tampered with, the proponent of the object must present evidence of chain of custody. The proponent must show that the object has been held in a substantially unbroken chain of possession. It is not necessary to negate all possibilities of substitution or tampering; rather, what is required is to show adherence to some system of identification and custody. Here, the proponent of the blood sample (the prosecution) has not shown what the officer did with it after leaving the crime scene. There is no showing that the vial was placed directly in a properly secured area so as to diminish the possibility of tampering. In short, it has not been demonstrated that there was adherence to some defined system of identification and custody. In the absence of a substantially unbroken chain of custody, the evidence is inadmissible for lack of proper authentication, and (A) is incorrect. (B) is incorrect. Although it may be true that the chemist qualifies as an expert witness (i.e., he has special knowledge, skill, experience, training, or education sufficient to qualify him as an expert on the subject to which his testimony relates), the fact remains that his testimony is inadmissible, as explained above. (D) is incorrect because the chemist would be permitted to testify to the results of the blood comparisons if there were proper authentication of the blood taken from the crime scene. There is no need for the chemist to have taken the original blood sample himself.
A retailer entered into an oral contract with an office supply wholesaler to buy 100 file boxes for an upcoming back to school sale at the retailer's store. The wholesaler agreed to deliver the file boxes in two weeks at a cost of $4 per file box. A week later, the retailer phoned the wholesaler and asked if she could increase her order to 200 file boxes. The wholesaler agreed. The wholesaler delivered the 200 file boxes as promised, but the retailer accepted only 150 upon discovering that she lacked storage space for all 200. May the wholesaler recover damages with respect to the 50 file boxes that were not accepted? response - incorrect A Yes, because the retailer accepted $600 worth of file boxes. B Yes, because the modification was for less than $500. C No, because the contract as modified was for $800. D No, because the wholesaler is a merchant with respect to file boxes.
The wholesaler may not recover damages. Under the UCC Statute of Frauds, a contract for the sale of goods for $500 or more is unenforceable unless evidenced by a writing signed by the party sought to be held liable. The original contract was for $400 and, thus, was not within the Statute. Whether a modification must be in writing to be enforceable depends on whether the entire contract price as modified is within the Statute. Here, the retailer and the wholesaler modified their original contract to 200 file boxes, bringing the total price to $800. Thus, the modification was unenforceable under the Statute of Frauds, and the wholesaler cannot collect damages with respect to the 50 unaccepted file boxes. (A) is incorrect. Acceptance is an exception to the Statute of Frauds—but only to the extent of the goods accepted. That is, an oral contract for the sale of goods for $500 or more is enforceable to the extent the goods are accepted. Here, the original contract for 100 boxes is enforceable, but the modification is not. However, since the retailer accepted 50 additional boxes, the modified contract is enforceable to the extent of the additional 50 boxes accepted. The fact that the accepted amount meets the Statute of Frauds $500 threshold does not make the contract enforceable for all 200. (B) is incorrect because, as noted above, when determining whether a contract for the sale of goods is enforceable, we look at the whole contract price as modified; the price of the modification itself does not matter. (D) is incorrect because the fact that the wholesaler is a merchant with respect to the goods being sold (file boxes) has no bearing on the enforceability of the contract here.
A man and a woman were in a two-car traffic accident. Immediately after the accident, the man was treated by a physician at a nearby hospital's emergency room. The man later filed a negligence action against the woman in federal district court, seeking compensatory damages. The woman now seeks discovery regarding the emergency room physician's observations, opinions, and treatment of the man. Is the woman entitled to discovery regarding that information? response - incorrect A No, because such discovery is not relevant to the claim or defense of a party. B No, because the woman is not entitled to obtain discovery from persons who are not parties to the action. C Yes as to the physician's observation and treatment, but the physician's opinions are discoverable only if the man intends to call the physician as an expert witness at trial. D Yes, because the physician observed and treated the man and developed opinions about the man's injuries for purposes other than litigation or trial.
The woman is entitled to discovery regarding the physician's observations, opinions, and treatment of the man because the physician developed opinions about the man's injuries for purposes other than litigation or trial. (A) is incorrect because the physician's observations, opinions, and treatment of the man are relevant to the man's claims and the driver's defenses. (B) is incorrect because parties may obtain discovery from individuals with knowledge of any discoverable matter, not just parties to the action. (C) is incorrect because it is not applicable to the situation here, where the physician was not retained in anticipation of litigation or trial.
The defendant was on trial for murder. The defendant called a witness to testify to an alibi. On cross-examination of the witness, the prosecutor asked, "Weren't you on the jury that acquitted the defendant of another criminal charge?" What is the best reason for sustaining an objection to this question? response - incorrect A The question goes beyond the scope of direct examination. B The probative value of the answer would be substantially outweighed by its tendency to mislead. C The question is a leading question. D Prior jury service in a case involving a party renders the witness incompetent.
This question raises several different issues: competency of witnesses, use of leading questions on cross-examination, the proper scope of cross-examination, and the probative value/prejudicial impact balancing test. Through a process of elimination, (B) emerges as the correct answer. (D) is incorrect. Under the Federal Rules, virtually all witnesses with personal knowledge are competent to testify. [Fed. R. Evid. 601] A witness is not rendered incompetent simply by having served on a jury in a prior case involving a party to the current suit. Such prior jury service might render the witness's testimony unpersuasive, but it would not make it inadmissible. (C) is incorrect because ordinarily, leading questions are permitted on cross-examination. [Fed. R. Evid. 611(c)] The prosecutor's question is a leading question, but that is perfectly permissible, especially in a case like this, where the alibi witness is not "friendly" toward the prosecution. (A) is incorrect because cross-examination is generally limited in scope to the subject matter of the direct examination and matters affecting the credibility of the witness [Fed. R. Evid. 611(b)], and the prosecutor's question is, in a roundabout way, an attempt to impeach the witness's credibility. The implication behind the question is that if the witness had served on a jury that acquitted the defendant of another criminal charge, the witness would be inclined to think the defendant innocent of the pending charge. Alternatively, the implication behind the question could be that the witness is the kind of person who is "soft on crime" and for that reason is not a credible witness. In either event, because the question is an attempt to impeach the witness's testimony, it is within the proper scope of cross-examination. This leaves (B) as the remaining correct answer. (B) is not unquestionably correct, because the probative value/prejudicial impact balancing test found in Rule 403 is weighted heavily toward admission of evidence. For evidence to be excluded under this balancing test, its probative value must be substantially outweighed by its prejudicial impact. Nevertheless, in this case, a plausible reason for sustaining an objection to the prosecutor's question is that the probative value of the answer would be substantially outweighed by its tendency to mislead. The question and answer would inevitably let the jury know that the defendant had been previously charged with a crime. This information could be highly prejudicial to his defense. Because the question and answer have little probative value (the negative inferences pertaining to the witness's credibility being very weak), it is reasonable to sustain an objection to the question on the basis that its probative value is substantially outweighed by its prejudicial impact.
A proud grandfather who planned to take pictures of his grandson's graduation purchased a camera from a camera store. He used the camera on several occasions over the next few weeks without incident, but when he used it on the day before his grandson's graduation, it caught fire and exploded, burning him and destroying an expensive coat he was wearing. Although the grandfather was in a great deal of pain because of his injuries, he insisted on attending his grandson's graduation. However, because he no longer had a workable camera, the grandfather hired a professional photographer to take pictures of the special day. In a breach of warranty action, which of the following represents the most that the grandfather may recover? response - incorrect A The difference between the value of the camera accepted and its value if it had been as warranted. B The difference between the value of the camera accepted and its value if it had been as warranted, plus medical costs for treating the grandfather's burns. C The difference between the value of the camera accepted and its value if it had been as warranted, medical costs for treating the grandfather's burns, and the cost to replace the grandfather's coat. D The difference between the value of the camera accepted and its value if it had been as warranted, medical costs for treating the grandfather's burns, the cost to replace the grandfather's coat, and the cost of hiring the professional photographer.
When a buyer accepts goods that turn out to be defective, he may recover as damages any "loss resulting in the normal course of events from the breach," which includes the difference between the value of the goods accepted and the value they would have had if they had been as warranted, plus incidental and consequential damages. Incidental damages resulting from the seller's breach include expenses reasonably incurred in inspection, receipt, and transportation, care, and custody of goods rightfully rejected. In this case, the grandfather incurred no incidental damages. Consequential damages resulting from the seller's breach include any loss resulting from general or particular requirements and needs of which the seller at the time of contracting had reason to know and which could not reasonably be prevented by cover or otherwise, and injury to person or property proximately resulting from any breach of warranty. Here, the grandfather is entitled to breach of warranty damages for the loss of the camera—the difference between the value of the camera accepted and its value if it had been as warranted—plus damages for injury to his person (e.g., medical costs for treating the grandfather's burns) and property (i.e., the cost to replace his coat) because they were proximately caused from the breach of warranty. Thus, (C) is correct, and (A) and (B) are wrong. (D) is wrong because the cost of hiring the professional photographer was not foreseeable. The seller was not told of any particular requirements and needs of the grandfather at the time of contracting nor would the seller have reason to know that the grandfather planned to use the camera to take pictures of his grandson's graduation and would hire a professional photographer if he lost the use of the camera.